[obm-l] Quadilátero

2018-09-06 Por tôpico Eduardo Wilner
a) Construir um quadrilátero convexo cujos lados opostos sejam diâmetros de 
circunferências que se tangenciam,     duas à duas, no interior do polígono.b) 
Calcular a distância entre os pontos de tangencia, em função do perímetro.   

-- 
Esta mensagem foi verificada pelo sistema de antiv�rus e
 acredita-se estar livre de perigo.



[obm-l] Constução geométrica livre

2018-01-29 Por tôpico Eduardo Wilner
Construir um triangulo isoceles ABC de base AC=2sqrt[6], cuja 
ceviana AD forma um ângulo de 15° com a base, sendo D um ponto 
do lado BC com BD=2.

-- 
Esta mensagem foi verificada pelo sistema de antiv�rus e
 acredita-se estar livre de perigo.



[obm-l] solução de questões da OBM

2017-11-09 Por tôpico Eduardo Wilner
Alguém sabe como se pode acessar solução (gabarito...) da 3ª fase (Nivel 2) da 
OBM, de 2015 por exemplo?Obrigado.


-- 
Esta mensagem foi verificada pelo sistema de antiv�rus e
 acredita-se estar livre de perigo.



[obm-l] Re: Questão de vetores

2016-09-18 Por tôpico Eduardo Wilner
Não entrou na minha caixa postal (e-mail)  e tbm não conseguí responder 
diretamente da lista. Vejamos se assim "funciona"...
 Sejam BM=x; MC=y, logo 3x=5y ou y=x*3/5 e BC=x+y=x*8/5 logo x=BM=BC*5/8. 

-- 
Esta mensagem foi verificada pelo sistema de antiv�rus e
 acredita-se estar livre de perigo.



[obm-l] Retângulo com "cúpula".

2016-09-17 Por tôpico Eduardo Wilner
ABCD é um retângulo tal que BC=AB.cos 45° e P é um ponto da 
semi-circunferência de diâmetro AB, externo ao retângulo e mais
próximo à B; sejam K e L as intersecções de AB com PD e PC, 
respectivamente.
  Q é outro ponto da semi-circunferência (vizinho à P) tal que 
AQ=AL. Se os prolongamentos de QK e QL encontram, respectivamente, 
CD em G e CB em H, provar que GAQH é um paralelograma.

-- 
Esta mensagem foi verificada pelo sistema de antiv�rus e
 acredita-se estar livre de perigo.



[obm-l] Re: [obm-l] Re: [obm-l] Re: [obm-l] RE: [obm-l] Pentágono regular

2014-01-24 Por tôpico Eduardo Wilner
Pentágono ABCDE. 

Caso I. Todos ângulos congruentes sucessivos (seja A =  B  =  C)

AC=AB=DB (*) pois os triângulos ABC, BCD e ABE são congruentes.
Assim, são congruentes os ângulos (1)  BCA = BEA (= CDB (**));

Portanto, os triângulos BCE e ACE são congruentes, logo 
(2)  ACE = CEB .

O triângulo CDE é isoceles, implicando em (3) ECD = DEC .

Somando membro a membro (1)+(2)+(3) obtemos C = E.

De (*) o triângulo BDE é isoceles logo BDE = DEB que somado membro a membro 
com CDB = BEA de (*) nos dá D = E


Caso II. Ângulos congruentes não todos sucessivos (seja E = A = C).

Análogamente ao Caso I  EB=AD=BD e(***) EAD = DBC (por congruência dos 
triângulos isoceles que os tem como base). 
Triângulo isoceles ABD =  BAD = ABD e somando membro a membro com (***) =  
A = B. 

Triângulo isoceles EBD = EDB = BED, somado com BDC = BEA dá

D = E.


[ ]s

       





Em Quinta-feira, 23 de Janeiro de 2014 17:25, Eduardo Wilner 
eduardowil...@yahoo.com.br escreveu:
 
Não seria conveniente especificar que o pentágono é convexo (caso contrário a 
afirmação não seria válida)?

[ ]s





Em Quarta-feira, 22 de Janeiro de 2014 9:15, luiz silva 
luizfelipec...@yahoo.com.br escreveu:
 
Sugestão : coloque o pentágono dentro de uma circunferência, e veja o que 
acontece com os outros ângulos (quadriláteros inscritiveis).

Abs
Felipe





Em Terça-feira, 21 de Janeiro de 2014 23:05, marcone augusto araújo borges 
marconeborge...@hotmail.com escreveu:
 
Eu tentei mais algumas vezes e não consegui.Peço ajuda.
 



From: marconeborge...@hotmail.com
To: obm-l@mat.puc-rio.br
Subject: [obm-l] Pentágono regular
Date: Sat, 18 Jan 2014 18:58:08 +

 
Prove que um pentágono de lados congruentes e 3 ângulos congruentes é regular 
-- 
Esta mensagem foi verificada pelo sistema de antivírus e 
acredita-se estar livre de perigo.

-- 
Esta mensagem foi verificada pelo sistema de antivírus e 
acredita-se estar livre de perigo. 


-- 
Esta mensagem foi verificada pelo sistema de antivírus e 
acredita-se estar livre de perigo. 


-- 
Esta mensagem foi verificada pelo sistema de antivírus e 
acredita-se estar livre de perigo. 
-- 
Esta mensagem foi verificada pelo sistema de antivírus e
 acredita-se estar livre de perigo.



[obm-l] Re: [obm-l] Re: [obm-l] RE: [obm-l] Pentágono regular

2014-01-23 Por tôpico Eduardo Wilner
Não seria conveniente especificar que o pentágono é convexo (caso contrário a 
afirmação não seria válida)?

[ ]s





Em Quarta-feira, 22 de Janeiro de 2014 9:15, luiz silva 
luizfelipec...@yahoo.com.br escreveu:
 
Sugestão : coloque o pentágono dentro de uma circunferência, e veja o que 
acontece com os outros ângulos (quadriláteros inscritiveis).

Abs
Felipe





Em Terça-feira, 21 de Janeiro de 2014 23:05, marcone augusto araújo borges 
marconeborge...@hotmail.com escreveu:
 
Eu tentei mais algumas vezes e não consegui.Peço ajuda.
 



From: marconeborge...@hotmail.com
To: obm-l@mat.puc-rio.br
Subject: [obm-l] Pentágono regular
Date: Sat, 18 Jan 2014 18:58:08 +

 
Prove que um pentágono de lados congruentes e 3 ângulos congruentes é regular 
-- 
Esta mensagem foi verificada pelo sistema de antivírus e 
acredita-se estar livre de perigo.

-- 
Esta mensagem foi verificada pelo sistema de antivírus e 
acredita-se estar livre de perigo. 


-- 
Esta mensagem foi verificada pelo sistema de antivírus e 
acredita-se estar livre de perigo. 
-- 
Esta mensagem foi verificada pelo sistema de antivírus e
 acredita-se estar livre de perigo.



Re: [obm-l] Como eu resolvo isso?

2014-01-06 Por tôpico Eduardo Wilner
Maldonado, poderia nos mostrar o problema original, de cinemática?

[ ]'s  





Em Sábado, 4 de Janeiro de 2014 18:08, saulo nilson saulo.nil...@gmail.com 
escreveu:
 
essa integral e catalogada caiu em uma prova da obmu.



2014/1/4 saulo nilson saulo.nil...@gmail.com

d2R/R=2d2acosa/sena
lnR dR=2(lnsena+V/D)da
RlnR-R+D=2aV/D+2Integral (lnsenada)



2014/1/3 João Maldonado joao_maldona...@hotmail.com

Fala ai galera. Eu tava resolvendo um problema de cinemática (sei que não é o 
assunto da lista) mas caí numa parte puramente matemática que não estou 
conseguindo resolver, queria pedir a ajuda de vocês. Se alguém puder me dar 
uma mão eu agradeço muito


d²R/dt² = 2cos(a)/R³
R d²a/dt² = sen(a)/R³


onde: R inicial = D
a inicial = 0
dR/dt inicial = 0
da/dt inicial = V/D


Quero achar R(t), a(t) e R(a)


[]'s
João
-- 
Esta mensagem foi verificada pelo sistema de antivírus e 
acredita-se estar livre de perigo. 



-- 
Esta mensagem foi verificada pelo sistema de antivírus e 
acredita-se estar livre de perigo. 
-- 
Esta mensagem foi verificada pelo sistema de antivírus e
 acredita-se estar livre de perigo.



Re: [obm-l] Coordenadas polares

2013-11-30 Por tôpico Eduardo Wilner
A secante é negativa no segundo e terceiro quadrante e rhô não pode ser 
negativo.

[ ]'s 




Em Sábado, 30 de Novembro de 2013 0:06, marcone augusto araújo borges 
marconeborge...@hotmail.com escreveu:
 
Eu nao entendi bem essa variação de teta.




Date: Sun, 24 Nov 2013 06:56:28 -0800
From: eduardowil...@yahoo.com.br
Subject: Re: [obm-l] Coordenadas polares
To: obm-l@mat.puc-rio.br


Como rô=0 , -pi/2= teta = pi/2 (à menos de k2pi);

Assim rô*cos(teta) = 1, que é a projeção de rô no eixo polar... tá enxergando?


[ ]'s




Em Domingo, 24 de Novembro de 2013 10:26, Bernardo Freitas Paulo da Costa 
bernardo...@gmail.com escreveu:
 
2013/11/24 marcone augusto araújo borges marconeborge...@hotmail.com

 Alguém poderia mostrar como fica o gráfico de rô = sec(teta)?

Transforme r e teta em coordenadas cartesianas, r^2 = x^2 + y^2,
tan(teta) = y/x, e depois substitua um pouco de trigonometria.

Abraços,
-- 
Bernardo Freitas Paulo da Costa


-- 
Esta mensagem foi verificada pelo sistema de antivírus e
acredita-se estar livre de perigo.


=
Instruções para entrar na lista, sair da lista e usar a lista em
http://www.mat.puc-rio.br/~obmlistas/obm-l.html

=



-- 
Esta mensagem foi verificada pelo sistema de antivírus e 
acredita-se estar livre de perigo.
-- 
Esta mensagem foi verificada pelo sistema de antivírus e 
acredita-se estar livre de perigo. 
-- 
Esta mensagem foi verificada pelo sistema de antivírus e
 acredita-se estar livre de perigo.



Re: [obm-l] Coordenadas polares

2013-11-24 Por tôpico Eduardo Wilner
Como rô=0 , -pi/2= teta = pi/2 (à menos de k2pi);

Assim rô*cos(teta) = 1, que é a projeção de rô no eixo polar... tá enxergando?


[ ]'s




Em Domingo, 24 de Novembro de 2013 10:26, Bernardo Freitas Paulo da Costa 
bernardo...@gmail.com escreveu:
 
2013/11/24 marcone augusto araújo borges marconeborge...@hotmail.com

 Alguém poderia mostrar como fica o gráfico de rô = sec(teta)?

Transforme r e teta em coordenadas cartesianas, r^2 = x^2 + y^2,
tan(teta) = y/x, e depois substitua um pouco de trigonometria.

Abraços,
-- 
Bernardo Freitas Paulo da Costa


-- 
Esta mensagem foi verificada pelo sistema de antivírus e
acredita-se estar livre de perigo.


=
Instruções para entrar na lista, sair da lista e usar a lista em
http://www.mat.puc-rio.br/~obmlistas/obm-l.html

=
-- 
Esta mensagem foi verificada pelo sistema de antivírus e
 acredita-se estar livre de perigo.



Re: [obm-l] Coordenadas polares

2013-11-24 Por tôpico Eduardo Wilner
Ninguém achou a minha proposta mais simples? 





Em Domingo, 24 de Novembro de 2013 22:26, Bernardo Freitas Paulo da Costa 
bernardo...@gmail.com escreveu:
 
2013/11/24 marcone augusto araújo borges marconeborge...@hotmail.com:
 r = sec(teta)
 r^2 = [sec(teta)]^2 = 1 + [tg(teta)]^2
 x^2 + y^2 = 1 + (y/x)^2
 E dai?

Daí que

x^2 + y^2 = 1 + (y/x)^2 = (x^2 + y^2)/x^2, logo x^2 = 1 (pois r != 0,
já que a secante é sempre diferente de zero, logo podemos cortar x^2 +
y^2). Daí, x = 1 ou x = -1, e mais um pouco de trigonometria vai
mostrar que x = -1 é uma solução espúria, introduzida porque a gente
elevou ao quadrado (isso quase sempre acontece, e é sempre bom
verificar como... aqui, se fosse r = - sec(teta), teria dado a mesma
depois de elevar ao quadrado, portanto claro que uma das soluções tem
que ser desprezada). Isso quer dizer que x = 1, ou seja, e é equação
de uma reta vertical.


Abraços,
-- 
Bernardo Freitas Paulo da Costa

-- 
Esta mensagem foi verificada pelo sistema de antivírus e
acredita-se estar livre de perigo.


=
Instruções para entrar na lista, sair da lista e usar a lista em
http://www.mat.puc-rio.br/~obmlistas/obm-l.html
=
-- 
Esta mensagem foi verificada pelo sistema de antivírus e
 acredita-se estar livre de perigo.



[obm-l] Re: [obm-l] Cônicas

2013-11-24 Por tôpico Eduardo Wilner
Uma translação dos eixos de forma que a origem coincida com o ponto dado 
(10;25) deixa o trabalho de encontrar m(1;-29/4) mais agradável.



[ ]'s




Em Domingo, 24 de Novembro de 2013 8:26, marcone augusto araújo borges 
marconeborge...@hotmail.com escreveu:
 
 
Determinar as equações das retas tangentes à cônica x^2 + 4y^2 - 180 = 0
 
que passam pelo ponto (10,25)

Eu escrevi  y - 25 =m(x - 10) *
A ideia era substituir o valor de y em * na equação lá de cima e igualar delta 
a zero
para achar m,mas as contas ficaram enormes          
-- 
Esta mensagem foi verificada pelo sistema de antivírus e 
acredita-se estar livre de perigo. 
-- 
Esta mensagem foi verificada pelo sistema de antivírus e
 acredita-se estar livre de perigo.



[obm-l] Re: [obm-l] Re: [obm-l] Re: [obm-l] Re: [obm-l] Diferença de cubos

2013-11-11 Por tôpico Eduardo Wilner
Quando eu coloquei a errata achei que estivesse claro que eu já tinha 
entendido... Afinal, para bom entendedor meia palavra besta, ops, basta...

O que confundiu foi que como estava o segundo membro da igualdade era maior 
que o primeiro!

E a soma dos quadrados?


[]'s




Em Segunda-feira, 11 de Novembro de 2013 23:51, terence thirteen 
peterdirich...@gmail.com escreveu:
 
Isso mesmo, é só abrir:

(b+1)^3 - b^3 =3a^2+3a+1

3a^2+3a+1=b^2


3*(4a^2+4a+1)+1=4b^2
3(2a+1)^2+1=(2b)^2

Eu usei uma técnica de completar quadrados neste parêntese.

Eu multipliquei por 4 para deixar tudo par, mas se quiser, eis a forma fácil de 
entender:

3a^2+3a+1=b^2


a^2+a+1/3=b^2/3

a^2+2*1/2*a+(1/2)^2 - (1/2)^2 + 1/3=b^2/3


(a+1/2)^2 - 1/4 + 1/3=b^2/3


Tira o mínimo e voilà!















Em 11 de novembro de 2013 00:22, Eduardo Wilner eduardowil...@yahoo.com.br 
escreveu:

No segundo membro, onde se lê b, leia-se a ?






Em Segunda-feira, 11 de Novembro de 2013 0:07, Eduardo Wilner 
eduardowil...@yahoo.com.br escreveu:
 
Como se chega à   (2b)^2=3(2b+1)^2+1 ?


A propósito, a expressão parece estar incorreta.






Em Domingo, 10 de Novembro de 2013 19:01, terence thirteen 
peterdirich...@gmail.com escreveu:
 
Isto equivale a uma equação de Pell!


(a+1)^3-a^3=b^2 acarreta
(2b)^2=3(2b+1)^2+1


Talvez usando reciprocidade, fique mais fácil...



Em 9 de novembro de 2013 23:20, marcone augusto araújo borges 
marconeborge...@hotmail.com escreveu:

Mostre que se a diferença de dois cubos consecutivos é um quadrado,então   
  
é o quadrado da soma de dois quadrados.
8^3 - 7^3 = (3^2 + 2^2)^2
-- 
Esta mensagem foi verificada pelo sistema de antivírus e 
acredita-se estar livre de perigo. 



-- 
/**/
神が祝福

Torres 
-- 
Esta mensagem foi verificada pelo sistema de antiv�us e 
acredita-se estar livre de perigo. 





-- 
Esta mensagem foi verificada pelo sistema de antivírus e 
acredita-se estar livre de perigo. 


-- 
/**/
神が祝福

Torres 
-- 
Esta mensagem foi verificada pelo sistema de antiv�us e 
acredita-se estar livre de perigo. 
-- 
Esta mensagem foi verificada pelo sistema de antiv�rus e
 acredita-se estar livre de perigo.



[obm-l] Re: [obm-l] Re: [obm-l] Re: [obm-l] Re: [obm-l] Diferença de cubos

2013-11-11 Por tôpico Eduardo Wilner
O que confundiu foi que, como estava, o segundo membro da igualdade era maior 
que o primeiro!




Em Terça-feira, 12 de Novembro de 2013 1:15, Eduardo Wilner 
eduardowil...@yahoo.com.br escreveu:
 
Quando eu coloquei a errata achei que estivesse claro que eu já tinha 
entendido... Afinal, para bom entendedor meia palavra besta, ops, basta...

O que confundiu foi que como estava o segundo membro da igualdade era maior 
que o primeiro!

E a soma dos quadrados?


[]'s




Em Segunda-feira, 11 de Novembro de 2013 23:51, terence thirteen 
peterdirich...@gmail.com escreveu:
 
Isso mesmo, é só abrir:

(b+1)^3 - b^3 =3a^2+3a+1

3a^2+3a+1=b^2


3*(4a^2+4a+1)+1=4b^2
3(2a+1)^2+1=(2b)^2

Eu usei uma técnica de completar quadrados neste parêntese.

Eu multipliquei por 4 para deixar tudo par, mas se quiser, eis a forma fácil de 
entender:

3a^2+3a+1=b^2


a^2+a+1/3=b^2/3

a^2+2*1/2*a+(1/2)^2 - (1/2)^2 + 1/3=b^2/3


(a+1/2)^2 - 1/4 + 1/3=b^2/3


Tira o mínimo e voilà!















Em 11 de novembro de 2013 00:22, Eduardo Wilner eduardowil...@yahoo.com.br 
escreveu:

No segundo membro, onde se lê b, leia-se a ?






Em Segunda-feira, 11 de Novembro de 2013 0:07, Eduardo Wilner 
eduardowil...@yahoo.com.br escreveu:
 
Como se chega à   (2b)^2=3(2b+1)^2+1 ?


A propósito, a expressão parece estar incorreta.






Em Domingo, 10 de Novembro de 2013 19:01, terence thirteen 
peterdirich...@gmail.com escreveu:
 
Isto equivale a uma equação de Pell!


(a+1)^3-a^3=b^2 acarreta
(2b)^2=3(2b+1)^2+1


Talvez usando reciprocidade, fique mais fácil...



Em 9 de novembro de 2013 23:20, marcone augusto araújo borges 
marconeborge...@hotmail.com escreveu:

Mostre que se a diferença de dois cubos consecutivos é um quadrado,então   
  
é o quadrado da soma de dois quadrados.
8^3 - 7^3 = (3^2 + 2^2)^2
-- 
Esta mensagem foi verificada pelo sistema de antivírus e 
acredita-se estar livre de perigo. 



-- 
/**/
神が祝福

Torres 
-- 
Esta mensagem foi verificada pelo sistema de antiv�us e 
acredita-se estar livre de perigo. 





-- 
Esta mensagem foi verificada pelo sistema de antivírus e 
acredita-se estar livre de perigo. 


-- 
/**/
神が祝福

Torres 
-- 
Esta mensagem foi verificada pelo sistema de antiv�us e 
acredita-se estar livre de perigo. 
-- 
Esta mensagem foi verificada pelo sistema de antiv�rus e
 acredita-se estar livre de perigo.



[obm-l] Re: [obm-l] Re: [obm-l] Diferença de cubos

2013-11-10 Por tôpico Eduardo Wilner
Como se chega à   (2b)^2=3(2b+1)^2+1 ?

A propósito, a expressão parece estar incorreta.





Em Domingo, 10 de Novembro de 2013 19:01, terence thirteen 
peterdirich...@gmail.com escreveu:
 
Isto equivale a uma equação de Pell!

(a+1)^3-a^3=b^2 acarreta
(2b)^2=3(2b+1)^2+1

Talvez usando reciprocidade, fique mais fácil...



Em 9 de novembro de 2013 23:20, marcone augusto araújo borges 
marconeborge...@hotmail.com escreveu:

Mostre que se a diferença de dois cubos consecutivos é um quadrado,então
 
é o quadrado da soma de dois quadrados.
8^3 - 7^3 = (3^2 + 2^2)^2
-- 
Esta mensagem foi verificada pelo sistema de antivírus e 
acredita-se estar livre de perigo. 


-- 
/**/
神が祝福

Torres 
-- 
Esta mensagem foi verificada pelo sistema de antiv�us e 
acredita-se estar livre de perigo. 
-- 
Esta mensagem foi verificada pelo sistema de antiv�rus e
 acredita-se estar livre de perigo.



[obm-l] Re: [obm-l] Re: [obm-l] Diferença de cubos

2013-11-10 Por tôpico Eduardo Wilner
No segundo membro, onde se lê b, leia-se a ?





Em Segunda-feira, 11 de Novembro de 2013 0:07, Eduardo Wilner 
eduardowil...@yahoo.com.br escreveu:
 
Como se chega à   (2b)^2=3(2b+1)^2+1 ?

A propósito, a expressão parece estar incorreta.





Em Domingo, 10 de Novembro de 2013 19:01, terence thirteen 
peterdirich...@gmail.com escreveu:
 
Isto equivale a uma equação de Pell!

(a+1)^3-a^3=b^2 acarreta
(2b)^2=3(2b+1)^2+1

Talvez usando reciprocidade, fique mais fácil...



Em 9 de novembro de 2013 23:20, marcone augusto araújo borges 
marconeborge...@hotmail.com escreveu:

Mostre que se a diferença de dois cubos consecutivos é um quadrado,então
 
é o quadrado da soma de dois quadrados.
8^3 - 7^3 = (3^2 + 2^2)^2
-- 
Esta mensagem foi verificada pelo sistema de antivírus e 
acredita-se estar livre de perigo. 


-- 
/**/
神が祝福

Torres 
-- 
Esta mensagem foi verificada pelo sistema de antiv�us e 
acredita-se estar livre de perigo. 
-- 
Esta mensagem foi verificada pelo sistema de antiv�rus e
 acredita-se estar livre de perigo.



Re: [obm-l] comprimento de corda elicoidal

2013-11-07 Por tôpico Eduardo Wilner
O comprimento depende do passo da hélice que não foi especificado.
Para liberá-lo podemos acrescentar um fator q ao argumento das funções 
trigonométricas; assim, para q=2pi, p.ex., a hélice daria uma volta completa,

[ ]' s




Em Quinta-feira, 7 de Novembro de 2013 1:04, Eduardo Wilner 
eduardowil...@yahoo.com.br escreveu:
 
Parece ser uma hélice conica que pode ser expressa paramétricamente como

f(t)=[ t.cos t, t.sen t, e.(t-1)]  , supondo que o 2,7 seja o neperiano e.

 Daí é só integrar |f'(t)|.dt entre t=1 e t=2.


[ ]'s  




Em Quarta-feira, 6 de Novembro de 2013 11:01, claudinei claudin...@gmail.com 
escreveu:
 

Alguem por favor poderia me ajudar respondendo como calculo o comprimento
 de uma corda elicoidal de raio variável? Exemplo: uma corda elicoidal de raio 
da base 1 e raio superior 2 de altura 2,7?
 
Desde ja agradeço


-- 
Esta mensagem foi verificada pelo sistema de antivírus e 
acredita-se estar livre de perigo. 


-- 
Esta mensagem foi verificada pelo sistema de antivírus e 
acredita-se estar livre de perigo. 
-- 
Esta mensagem foi verificada pelo sistema de antivírus e
 acredita-se estar livre de perigo.



Re: [obm-l] comprimento de corda elicoidal

2013-11-06 Por tôpico Eduardo Wilner
Parece ser uma hélice conica que pode ser expressa paramétricamente como

f(t)=[ t.cos t, t.sen t, e.(t-1)]  , supondo que o 2,7 seja o neperiano e.

 Daí é só integrar |f'(t)|.dt entre t=1 e t=2.


[ ]'s  




Em Quarta-feira, 6 de Novembro de 2013 11:01, claudinei claudin...@gmail.com 
escreveu:
 

Alguem por favor poderia me ajudar respondendo como calculo o comprimento
 de uma corda elicoidal de raio variável? Exemplo: uma corda elicoidal de raio 
da base 1 e raio superior 2 de altura 2,7?
 
Desde ja agradeço


-- 
Esta mensagem foi verificada pelo sistema de antivírus e 
acredita-se estar livre de perigo. 
-- 
Esta mensagem foi verificada pelo sistema de antivírus e
 acredita-se estar livre de perigo.



[obm-l] O regresso da Elipse e círculo

2013-10-10 Por tôpico Eduardo Wilner
Há alguns dias Douglas Oliveira manifestou sua surpresa quando não conseguiu 
resolver o problema de tangencia usando geometria analítica e impondo 
discriminante nulo na equação da intersecção, e na ocasião alertei que o motivo 
era ter sido escolhido o foco para centro da circunferencia.

Na verdade, o foco não é o unico ponto que gera este problema.

Me parece interessante propor a questão: que pontos do (eixo maior da elipse) 
servem como centro, para que o método tentado por Douglas funcione? 



[ ]'s    

-- 
Esta mensagem foi verificada pelo sistema de antivírus e
 acredita-se estar livre de perigo.



Re: [obm-l] Quadrado perfeito

2013-10-08 Por tôpico Eduardo Wilner


(3x-4)^2 - 4 = n^2   , se m = 3x -4   = m^2 - n^2 = 4   ou  (m/2)^2  - (n/2)^2 
=1

Equação de Pell com parâmetro , 1, quadrado perfeito .   


Assim n=0  e m/2 = + ou - 1 = 3x -4 = + ou - 2   = x = 2 (ou 2/3 que não é 
inteiro).

[ ]'s
De: marcone augusto araújo borges marconeborge...@hotmail.com
Para: obm-l@mat.puc-rio.br obm-l@mat.puc-rio.br 
Enviadas: Terça-feira, 8 de Outubro de 2013 9:15
Assunto: [obm-l] Quadrado perfeito
 


 
Determine todos os valores inteiros positivos de x tais que 9x^2 - 24x + 12 

é um quadrado perfeito.
-- 
Esta mensagem foi verificada pelo sistema de antivírus e 
acredita-se estar livre de perigo. 
-- 
Esta mensagem foi verificada pelo sistema de antivírus e
 acredita-se estar livre de perigo.



[obm-l] Re: [obm-l] Elipse e círculo

2013-10-02 Por tôpico Eduardo Wilner
É um caso particular pelo fato da circunferência estar centrada no foco.

Observe que o ponto de tangência utilizando o raio encontrado com  geometria 
plana é o vértice 
da elipse, e lembre-se que a normal à elipse é bissetriz do ângulo com vértice 
no ponto da elise e com lados passando pelos focos.

[ ]'s  



 De: douglas.olive...@grupoolimpo.com.br douglas.olive...@grupoolimpo.com.br
Para: obm-l@mat.puc-rio.br 
Enviadas: Quarta-feira, 2 de Outubro de 2013 13:57
Assunto: [obm-l] Elipse e círculo
 


Caros amigos elaborei um exercício em sala de aula, com a intenção de resolver 
mais um problema de tangencia,
peguei uma elipse com eixo maior igual a 10cm e eixo menor igual a 6cm e nesta 
elipse inscrevi uma circunferência
com centro em um dos focos e fui calcular o raio desta circunferência para que 
ela fosse tangente á elipse
(por dentro) escrevi a equação da elipse e a equação da circunferência e usei a 
condição padrão de
tangencia( delta=zero) , mas para minha surpresa não deu muito certo e o raio 
fico igual a zero,
porém em outra solução (por geometria plana)
encontrei o raio igual a 1cm , e agora? Seria um caso particular? Alguém já 
passou por isso?
 
Att: Douglas Oliveira
 
 
 
-- 
Esta mensagem foi verificada pelo sistema de antiv�us e 
acredita-se estar livre de perigo. 
-- 
Esta mensagem foi verificada pelo sistema de antiv�rus e
 acredita-se estar livre de perigo.



[obm-l] Re: [obm-l] RE: [obm-l] Polinômios

2013-09-26 Por tôpico Eduardo Wilner
Oi Maldonado.

Gostaria de entender a notação:  

parece que cp seriam as raizes, mas, em cp=1/ap, ap seriam os coeficientes? 
Como?

[ ]'s





 De: João Maldonado joao_maldona...@hotmail.com
Para: obm-l@mat.puc-rio.br obm-l@mat.puc-rio.br 
Enviadas: Terça-feira, 24 de Setembro de 2013 23:00
Assunto: [obm-l] RE: [obm-l] Polinômios
 


 
Sendo cp = 1/ap
a1a2...an =  +-1/an
a1a2...an(1/a1 + 1/a2+...+1/an) =   -+1/an
a1a2...an(1/a1a2 + 1/a1a3 +... +1/an-1an) =   +-1/an

Logo: (1/a1 + 1/a2+...+1/an) = -1
(1/a1a2 + 1/a1a3 +... +1/an-1an) =1
x=c1+c2+ ... +cn = -1
y=c1c2+c1c3+...+cn-1cn = 1

c1²+c2² +... +cn² = (c1+c2+ ... +cn)² -2(c1c2+c1c3+...+cn-1cn) = -1, absurdo, 
logo para n par temos  que pelo menos 2 raízes são complexas

[]'s
João





From: marconeborge...@hotmail.com
To: obm-l@mat.puc-rio.br
Subject: [obm-l] Polinômios 
Date: Wed, 25 Sep 2013 01:00:58 +

 
Prove que um polinômio do tipo a(n)x^n + a(n-1)x^(n-1) + ...+ a(3)x^3 + x^2 + x 
+ 1 com 
coeficientes reais não pode ter todas as raízes reais.

-- 
Esta mensagem foi verificada pelo sistema de antivírus e 
acredita-se estar livre de perigo.
-- 
Esta mensagem foi verificada pelo sistema de antivírus e 
acredita-se estar livre de perigo. 
-- 
Esta mensagem foi verificada pelo sistema de antiv�rus e
 acredita-se estar livre de perigo.



Re: [obm-l] Como que faz??

2013-09-25 Por tôpico Eduardo Wilner
Oi Carlos.

No item 2) vc. diz que CGE = x+y; isto significaria,  CGE = ADE . Vc. 
poderia explicar?

Obrigado

[ ]'s





 De: Carlos Victor victorcar...@globo.com
Para: obm-l@mat.puc-rio.br 
Enviadas: Terça-feira, 24 de Setembro de 2013 19:30
Assunto: Re: [obm-l] Como que faz??
 


Olá Douglas,
Acredito ter conseguido uma resolução para o problema 2 de geometria que vc 
postou aqui  .

Vamos lá  e acompanhe fazendo a figura , ok ?
vamos provar que na verdade o ângulo DEF é o dobro de ADC.
Seja  o ângulo  ADC = x e o ângulo CDE = y .

1) Trace CE e observe que o quadrilátero ACED é inscritível . então AEC = x  e  
EAC = y .

2) seja G a intersecção de CD com a circunferência C´ . Trace  EG e observe que 
 o ângulo CGE = x + y . Daí concluímos que o ângulo GED = x .

3) Não é difícil de mostrar que EB  é bissetriz de AEG . Seja então os ângulos 
AEB= DEB = z .

4) Trace agora a perpendicular de B  ao segmento ED e seja H o pé desta 
perpendicular. Observe que o quadrilátero BFHD é inscritível , então BHF = x 
.Trace FH e observe que EG é perpendicular  a FH . Seja J a intersecção de FH 
com EG .

5) Como o triângulo CEG está inscrito na circunferência C´ e observando que BF 
é perpendicular ao lado CD , pelo enunciado ; teremos pela reta de SIMSON , que 
 os pés das  perpendiculares traçadas de B aos lados CG , EG e EC  estão 
alinhados. Sejam então I o pé da perpendicular traçada de ao lado EG  e R o pé  
da perpendicular traçada de B ao lado CE .

6) observando os quadriláteros inscritíveis : BIER , BIHE , teremos q










Em 23 de agosto de 2013 16:03, douglas.olive...@grupoolimpo.com.br escreveu:

 
Olá , alguns alunos do ensino médio da instituição onde trabalho me deram 
alguns problemas do site https://brilliant.org/
E não consegui achar solução para dois deles, vou escreve-los abaixo e se 
alguém puder me ajudar agradeço.
 
PROBLEMA 1: Dada uma função f:R-R tal que f(2x^2 -1)=2(f(x))^2 -1 e f(x) é um 
polinômio de grau 13, sendo assim determine o coeficiente de x^5 de f(x).
 
PROBLEMA 2: Seja uma circunferência C'  e um ponto externo A , traça-se por A 
duas tangentes a circunferência que a interceptam nos pontos B e C , marca-se 
no prolongamento de AB no sentido de A para B um ponto D tal que o ângulo 
ADC=25 graus, traça-se por B uma perpendicular ao segmento CD que intercepta 
CD em F . Agora considere um outra circunferência C'' circunscrita ao 
triângulo ADC que intercepta a primeira circunferência C' no ponto E . 
Determinar a medida do ângulo DEF.
 
 
Obs: Fiz a segunda figura no geogebra e encontrei 50 graus como resposta , 
preciso na verdade de uma resolução. 
 
Att, Douglas Oliveira. 
-- 
Esta mensagem foi verificada pelo sistema de antivírus e 
acredita-se estar livre de perigo. 

-- 
Esta mensagem foi verificada pelo sistema de antivírus e 
acredita-se estar livre de perigo. 
-- 
Esta mensagem foi verificada pelo sistema de antivírus e
 acredita-se estar livre de perigo.



[obm-l] Re: [obm-l] Re: [obm-l] Re: [obm-l] Álgebra(não tá saindo)

2013-09-18 Por tôpico Eduardo Wilner
Não seria n = 5 + 6k ? Apenas para fins de clareza e precisão, pois no caso 
interessa apenas o mínimo (5)...

[ ]'s 





 De: Willy George Amaral Petrenko wgapetre...@gmail.com
Para: obm-l@mat.puc-rio.br 
Enviadas: Sábado, 14 de Setembro de 2013 23:34
Assunto: [obm-l] Re: [obm-l] Re: [obm-l] Álgebra(não tá saindo)
 


Ou resolva a equação em N:

(10*x+6)*4 = 6*10n + x ⇒ 39*x + 24 = 6*10n ⇒ 13*x = 2*10n - 8 ⇒ 10n = 4 mod 13 
⇒ n = 5 + 12k. Logo o menor n é 5 e o menor número é (2*105 - 8)/13 = 15384  
Obviamente vc adiciona o 6 depois: 153846



2013/9/14 Ralph Teixeira ralp...@gmail.com

Escreva a multiplicacao que nem a gente fazia lah na 4a serie:
_6
              x4
6_


Agora vah fazendo a multiplicacao. 6x4=24, entao poe o 4, vai 2.
Mas, se eh 4 ali embaixo, eh 4 do lado esquero do 6. Entao fica algo assim:
46
               x4
64Agora 4x4=16, mais 2, dah 18. Entao poe o 8 no resultado E TAMBEM DO 
LADO DO 4 NA PRIMEIRA LINHA (e vai 1).
___846

               x4
6___84
4x8=32, +1=33. O proximo eh 3. Continue assim achando os digitos da direita 
para a esquerda: 5, 1... E entao o proximo eh 6, que PODE ser aquele 6 
inicial! 


Assim, o menor numero inteiro n eh 153846.


Abraco,
        Ralph





2013/9/14 marcone augusto araújo borges marconeborge...@hotmail.com

Encontre o menor inteiro positivo n que possui as seguintes propriedades:  
   
I. Em sua representação tem o 6 como último dígito
II.Se o último dígito(6) é apagado  e colocado na frente dos dígitos 
restantes,o número resultante
é quatro vezes maior que o número original n
-- 
Esta mensagem foi verificada pelo sistema de antivírus e 
acredita-se estar livre de perigo. 

-- 
Esta mensagem foi verificada pelo sistema de antivírus e 
acredita-se estar livre de perigo. 

-- 
Esta mensagem foi verificada pelo sistema de antivírus e 
acredita-se estar livre de perigo. 
-- 
Esta mensagem foi verificada pelo sistema de antiv�rus e
 acredita-se estar livre de perigo.



[obm-l] Re: [obm-l] Re: [obm-l] Re: [obm-l] Re: [obm-l] Álgebra(não tá saindo)

2013-09-16 Por tôpico Eduardo Wilner
x tem que ser par: seja x=2y = 10n = 13*y + 4 ...

[ ]'s





 De: Hermann ilhadepaqu...@bol.com.br
Para: obm-l@mat.puc-rio.br 
Enviadas: Domingo, 15 de Setembro de 2013 11:18
Assunto: [obm-l] Re: [obm-l] Re: [obm-l] Re: [obm-l] Álgebra(não tá saindo)
 


 
Poderiam me explicar essa passagem
 13*x = 2*10n - 8 ⇒ 10n = 4 mod 13 
obrigado
 Hermann
- Original Message - 
From: Willy  George Amaral Petrenko 
To: obm-l@mat.puc-rio.br 
Sent: Saturday, September 14, 2013 11:34  PM
Subject: [obm-l] Re: [obm-l] Re: [obm-l]  Álgebra(não tá saindo)


Ou resolva a equação em N: 


(10*x+6)*4 = 6*10n + x ⇒ 39*x + 24 = 6*10n ⇒ 13*x =  2*10n - 8 ⇒ 10n = 4 mod 
13 ⇒ n = 5 + 12k. Logo o menor n  é 5 e o menor número é (2*105 - 8)/13 = 
15384   Obviamente vc adiciona o 6 depois: 153846



2013/9/14 Ralph Teixeira ralp...@gmail.com

Escreva a multiplicacao que nem a gente fazia lah na 4a serie:
_6
              x4
6_


Agora vah fazendo a multiplicacao. 6x4=24, entao poe o 4, vai 2.
Mas, se eh 4 ali embaixo, eh 4 do lado esquero do 6. Entao fica algo  assim:
46
               x4
64Agora 4x4=16, mais 2, dah 18. Entao poe o 8 no  resultado E TAMBEM 
DO LADO DO 4 NA PRIMEIRA LINHA (e vai 1). 
___846

               x4
6___84
4x8=32, +1=33. O proximo eh 3. Continue assim achando os digitos da  direita 
para a esquerda: 5, 1... E entao o proximo eh 6, que PODE ser aquele  6 
inicial!  


Assim, o menor numero inteiro n eh 153846.


Abraco,
        Ralph





2013/9/14 marcone augusto araújo borges marconeborge...@hotmail.com

Encontre o menor inteiro positivo n que possui as seguintes  propriedades:
  
I. Em sua representação tem o 6 como último dígito
II.Se o último dígito(6) é apagado  e colocado na frente dos  dígitos 
restantes,o número resultante
é quatro vezes maior que o número original n
-- 
Esta mensagem foi verificada pelo sistema de antivírus e 
acredita-se estar livre de perigo. 

-- 
Esta mensagem 
foi verificada pelo sistema de antivírus e 
acredita-se estar livre de 
perigo. 

-- 
Esta mensagem foi 
  verificada pelo sistema de antivírus e 
acredita-se estar livre de perigo. 
-- 
Esta mensagem foi verificada pelo sistema de antivírus e 
acredita-se estar livre de perigo. 
-- 
Esta mensagem foi verificada pelo sistema de antiv�rus e
 acredita-se estar livre de perigo.



[obm-l] Re: [obm-l] RE: [obm-l] RE: [obm-l] RE: [obm-l] RE: [obm-l] Ajuda em Geometria analítica

2013-09-12 Por tôpico Eduardo Wilner
Pode mas não é necessário,

Como Maldonado mostrou, ao longo do diâmetro ( de equação y/x=m) y/x é 
constante,   
portanto este quociente é o mesmonas extremidades do diâmetro.  




 De: marcone augusto araújo borges marconeborge...@hotmail.com
Para: obm-l@mat.puc-rio.br obm-l@mat.puc-rio.br 
Enviadas: Quinta-feira, 12 de Setembro de 2013 15:07
Assunto: [obm-l] RE: [obm-l] RE: [obm-l] RE: [obm-l] RE: [obm-l] Ajuda em 
Geometria analítica
 


 
Levando em conta que os pontos de intersecção são da forma (x,y) e 
(-x,-y),poderíamos
mostrar,usando a fórmula de distância de um ponto a uma reta,que as distâncias 
de cada um deles às retas tangentes(opostas)são iguais e dai concluir que essas 
retas tangentes são paralelas?




From: joao_maldona...@hotmail.com
To: obm-l@mat.puc-rio.br
Subject: [obm-l] RE: [obm-l] RE: [obm-l] RE: [obm-l] Ajuda em Geometria 
analítica
Date: Thu, 12 Sep 2013 13:18:17 -0300

 
O y/x é constante para os dois pontos de intersecção. Repare que temos 
infinitos m que satisfazem y=mx, mas cada diametro  da elipse é formado por uma 
unica reta (um unico m) que gera dois pontos de intereeccao distintos, porem 
nesses dois pontos o y/x é o mesmo   


From: marconeborge...@hotmail.com
To: obm-l@mat.puc-rio.br
Subject: [obm-l] RE: [obm-l] RE: [obm-l] Ajuda em Geometria analítica
Date: Thu, 12 Sep 2013 12:07:03 +

 
Desculpe,mas por que x/y é constante?




From: joao_maldona...@hotmail.com
To: obm-l@mat.puc-rio.br
Subject: [obm-l] RE: [obm-l] Ajuda em Geometria analítica 
Date: Thu, 12 Sep 2013 02:22:32 -0300

 
Seja a elipse centrada na origem x²/a² + y²/b² = 1
Derivando temos 2xdx/a² + 2ydy/b² = 0, dy/dx = (-x/y) (b²/a²)
Como a reta diametral é da forma y = mx, x/y é constante - dy/dx = constante 
- retas paralelas

[]s
João




From: marconeborge...@hotmail.com
To: obm-l@mat.puc-rio.br
Subject: [obm-l] Ajuda em Geometria analítica
Date: Thu, 12 Sep 2013 02:34:54 +

 
Prove que duas retas tangentes a uma elipse pelos pontos extremos de um 
 
diâmetro são paralelas.
-- 
Esta mensagem foi verificada pelo sistema de antivírus e 
acredita-se estar livre de perigo.
-- 
Esta mensagem foi verificada pelo sistema de antivírus e 
acredita-se estar livre de perigo.
-- 
Esta mensagem foi verificada pelo sistema de antivírus e 
acredita-se estar livre de perigo. 
-- 
Esta mensagem foi verificada pelo sistema de antivírus e 
acredita-se estar livre de perigo.
-- 
Esta mensagem foi verificada pelo sistema de antivírus e 
acredita-se estar livre de perigo. 
-- 
Esta mensagem foi verificada pelo sistema de antiv�rus e
 acredita-se estar livre de perigo.



[obm-l] Re: [obm-l] RE: [obm-l] RE: [obm-l] RE: [obm-l] RE: [obm-l] RE: [obm-l] Ajuda em Geometria analítica

2013-09-12 Por tôpico Eduardo Wilner
Claro que está correto; Seja P1 em uma reta e o pé da perpenciular à outra N1, 
P2 na outra reta 

com N2 pé da perpencidular à ptimeira reta.
P1 N1 P2 N2 representa um retângulo!

[]'s





 De: João Maldonado joao_maldona...@hotmail.com
Para: obm-l@mat.puc-rio.br 
Enviadas: Quinta-feira, 12 de Setembro de 2013 19:33
Assunto: [obm-l] RE: [obm-l] RE: [obm-l] RE: [obm-l] RE: [obm-l] RE: [obm-l] 
Ajuda em Geometria analítica
 


 
Eu nao vejo porque isso estaria certo, se tivermos duas retas, com um ponto em 
cada uma, tal que a distancia de cada um deles à reta oposta é a mesma, não 
quer dizer que as retas sejam paralelas


From: marconeborge...@hotmail.com
To: obm-l@mat.puc-rio.br
Subject: [obm-l] RE: [obm-l] RE: [obm-l] RE: [obm-l] RE: [obm-l] Ajuda em 
Geometria analítica
Date: Thu, 12 Sep 2013 18:07:49 +

 
Levando em conta que os pontos de intersecção são da forma (x,y) e 
(-x,-y),poderíamos
mostrar,usando a fórmula de distância de um ponto a uma reta,que as distâncias 
de cada um deles às retas tangentes(opostas)são iguais e dai concluir que essas 
retas tangentes são paralelas?




From: joao_maldona...@hotmail.com
To: obm-l@mat.puc-rio.br
Subject: [obm-l] RE: [obm-l] RE: [obm-l] RE: [obm-l] Ajuda em Geometria 
analítica
Date: Thu, 12 Sep 2013 13:18:17 -0300

 
O y/x é constante para os dois pontos de intersecção. Repare que temos 
infinitos m que satisfazem y=mx, mas cada diametro  da elipse é formado por uma 
unica reta (um unico m) que gera dois pontos de intereeccao distintos, porem 
nesses dois pontos o y/x é o mesmo   


From: marconeborge...@hotmail.com
To: obm-l@mat.puc-rio.br
Subject: [obm-l] RE: [obm-l] RE: [obm-l] Ajuda em Geometria analítica
Date: Thu, 12 Sep 2013 12:07:03 +

 
Desculpe,mas por que x/y é constante?




From: joao_maldona...@hotmail.com
To: obm-l@mat.puc-rio.br
Subject: [obm-l] RE: [obm-l] Ajuda em Geometria analítica 
Date: Thu, 12 Sep 2013 02:22:32 -0300

 
Seja a elipse centrada na origem x²/a² + y²/b² = 1
Derivando temos 2xdx/a² + 2ydy/b² = 0, dy/dx = (-x/y) (b²/a²)
Como a reta diametral é da forma y = mx, x/y é constante - dy/dx = constante 
- retas paralelas

[]s
João




From: marconeborge...@hotmail.com
To: obm-l@mat.puc-rio.br
Subject: [obm-l] Ajuda em Geometria analítica
Date: Thu, 12 Sep 2013 02:34:54 +

 
Prove que duas retas tangentes a uma elipse pelos pontos extremos de um 
 
diâmetro são paralelas.
-- 
Esta mensagem foi verificada pelo sistema de antivírus e 
acredita-se estar livre de perigo.
-- 
Esta mensagem foi verificada pelo sistema de antivírus e 
acredita-se estar livre de perigo.
-- 
Esta mensagem foi verificada pelo sistema de antivírus e 
acredita-se estar livre de perigo. 
-- 
Esta mensagem foi verificada pelo sistema de antivírus e 
acredita-se estar livre de perigo.
-- 
Esta mensagem foi verificada pelo sistema de antivírus e 
acredita-se estar livre de perigo. 
-- 
Esta mensagem foi verificada pelo sistema de antivírus e 
acredita-se estar livre de perigo. 
-- 
Esta mensagem foi verificada pelo sistema de antiv�rus e
 acredita-se estar livre de perigo.



[obm-l] Re: [obm-l] Re: [obm-l] Re: [obm-l] RE: [obm-l] Re: [obm-l] Teoria dos números

2013-08-30 Por tôpico Eduardo Wilner
Verdade! Obrigado!

[ ]'s





 De: Ralph Teixeira ralp...@gmail.com
Para: obm-l@mat.puc-rio.br 
Enviadas: Sexta-feira, 30 de Agosto de 2013 10:34
Assunto: [obm-l] Re: [obm-l] Re: [obm-l] RE: [obm-l] Re: [obm-l] Teoria dos 
números
 


Acho que voce pensou em 7^x como multiplicacao - ele quer potencia...:-( :-( 

On Aug 29, 2013 9:17 PM, Eduardo Wilner eduardowil...@yahoo.com.br wrote:

Observe que (1 + 3k , 1 + 7k) , k inteiro, satisfaz a equação diofantina

[ ]'s







 De: marcone augusto araújo borges marconeborge...@hotmail.com
Para: obm-l@mat.puc-rio.br obm-l@mat.puc-rio.br 
Enviadas: Quinta-feira, 29 de Agosto de 2013 12:18
Assunto: [obm-l] RE: [obm-l] Re: [obm-l] Teoria dos números
 


7^x modulo 9 dá 1,7 e 4 e 3^y dá quase sempre 0
O que interessa  para 7^x modulo 9 é 4,o que ocorre apenas quando x é da forma 
3.k + 2
Como x tambem é ímpar,só pode ser da forma 6.n + 5,mas...




Date: Thu, 29 Aug 2013 09:21:24 -0300
Subject: [obm-l] Re: [obm-l] Teoria dos números
From: ralp...@gmail.com
To: obm-l@mat.puc-rio.br


Tente agora modulo 9.
On Aug 28, 2013 9:50 PM, marcone augusto araújo borges 
marconeborge...@hotmail.com wrote:

Eu já postei a questão aqui,mas infelizmente não obtive resposta.
Sei que vão aparecendo outras questões interessantes e por isso peço licença 
para reapresentá-la
 
Determine todos os inteiros positivos x,y tais que 7^x - 3^y = 4


Claro que x = 1 e y = 1 satisfaz(desconfio que seja a unica solução)
Eu só consegui concluir que x e y são ímpares,analisando módulo 4.
Desde já agradeço.
 


-- 
Esta mensagem foi verificada pelo sistema de antivírus e 
acredita-se estar livre de perigo. 
-- 
Esta mensagem foi verificada pelo sistema de antivírus e 
acredita-se estar livre de perigo.
-- 
Esta mensagem foi verificada pelo sistema de antivírus e 
acredita-se estar livre de perigo. 


-- 
Esta mensagem foi verificada pelo sistema de antivírus e 
acredita-se estar livre de perigo. 
-- 
Esta mensagem foi verificada pelo sistema de antivírus e 
acredita-se estar livre de perigo. 
-- 
Esta mensagem foi verificada pelo sistema de antiv�rus e
 acredita-se estar livre de perigo.



[obm-l] Re: [obm-l] RE: [obm-l] Re: [obm-l] Teoria dos números

2013-08-29 Por tôpico Eduardo Wilner

Observe que (1 + 3k , 1 + 7k) , k inteiro, satisfaz a equação diofantina

[ ]'s





 De: marcone augusto araújo borges marconeborge...@hotmail.com
Para: obm-l@mat.puc-rio.br obm-l@mat.puc-rio.br 
Enviadas: Quinta-feira, 29 de Agosto de 2013 12:18
Assunto: [obm-l] RE: [obm-l] Re: [obm-l] Teoria dos números
 


 
7^x modulo 9 dá 1,7 e 4 e 3^y dá quase sempre 0
O que interessa  para 7^x modulo 9 é 4,o que ocorre apenas quando x é da forma 
3.k + 2
Como x tambem é ímpar,só pode ser da forma 6.n + 5,mas...




Date: Thu, 29 Aug 2013 09:21:24 -0300
Subject: [obm-l] Re: [obm-l] Teoria dos números
From: ralp...@gmail.com
To: obm-l@mat.puc-rio.br


Tente agora modulo 9.
On Aug 28, 2013 9:50 PM, marcone augusto araújo borges 
marconeborge...@hotmail.com wrote:

Eu já postei a questão aqui,mas infelizmente não obtive resposta.
Sei que vão aparecendo outras questões interessantes e por isso peço licença 
para reapresentá-la
 
Determine todos os inteiros positivos x,y tais que 7^x - 3^y = 4


Claro que x = 1 e y = 1 satisfaz(desconfio que seja a unica solução)
Eu só consegui concluir que x e y são ímpares,analisando módulo 4.
Desde já agradeço.
 


-- 
Esta mensagem foi verificada pelo sistema de antivírus e 
acredita-se estar livre de perigo. 
-- 
Esta mensagem foi verificada pelo sistema de antivírus e 
acredita-se estar livre de perigo.
-- 
Esta mensagem foi verificada pelo sistema de antivírus e 
acredita-se estar livre de perigo. 
-- 
Esta mensagem foi verificada pelo sistema de antiv�rus e
 acredita-se estar livre de perigo.



[obm-l] Re: [obm-l] Teoria dos números

2013-08-28 Por tôpico Eduardo Wilner
Observe que (1 + 3k , 1 + 7k) , k inteiro, satisfaz a equação diofantina

[ ]'s





 De: marcone augusto araújo borges marconeborge...@hotmail.com
Para: obm-l@mat.puc-rio.br obm-l@mat.puc-rio.br 
Enviadas: Quarta-feira, 28 de Agosto de 2013 21:45
Assunto: [obm-l] Teoria dos números
 


 
Eu já postei a questão aqui,mas infelizmente não obtive resposta.
Sei que vão aparecendo outras questões interessantes e por isso peço licença 
para reapresentá-la
 
Determine todos os inteiros positivos x,y tais que 7^x - 3^y = 4

Claro que x = 1 e y = 1 satisfaz(desconfio que seja a unica solução)
Eu só consegui concluir que x e y são ímpares,analisando módulo 4.
Desde já agradeço.
 

-- 
Esta mensagem foi verificada pelo sistema de antivírus e 
acredita-se estar livre de perigo. 
-- 
Esta mensagem foi verificada pelo sistema de antiv�rus e
 acredita-se estar livre de perigo.



Re: [obm-l] Como que faz??

2013-08-25 Por tôpico Eduardo Wilner
De uma ou de outra forma dá bastante equações mesmo! (seriam quatorze?)



[ ]'s




 De: Bernardo Freitas Paulo da Costa bernardo...@gmail.com
Para: obm-l@mat.puc-rio.br 
Enviadas: Sexta-feira, 23 de Agosto de 2013 19:45
Assunto: Re: [obm-l] Como que faz??
 

2013/8/23  douglas.olive...@grupoolimpo.com.br:
 Olá , alguns alunos do ensino médio da instituição onde trabalho me deram
 alguns problemas do site https://brilliant.org/

 PROBLEMA 1: Dada uma função f:R-R tal que f(2x^2 -1)=2(f(x))^2 -1 e f(x) é
 um polinômio de grau 13, sendo assim determine o coeficiente de x^5 de f(x).
Bom, f(x) é dada por 14 coeficientes a_i. A equação que você tem dá um
monte de condições sobre estes coeficientes: para cada x, tem uma
condição.

Por exemplo, x = 0 dá f(-1) = 2f(0)^2 - 1, f(-1) é uma soma alternada,
f(0)^2 é apenas (a_0)^2. Escolhendo um monte de x's, você obtém
bastante equações, e resolve.

Dá pra fazer (um pouco) menos grotesco, porque você pode escrever (a_0
+ a_1 * x + a_2 * x^2 +  + a_13 * x^13)^2, separando por grau.
Duas funções polinomiais em R são iguais se e somente se os
coeficientes forem iguais. Assim, identifique os coeficientes de ambos
os lados, e parta pro abraço.

Pra entender porque isso sempre dá certo, vale a pena lembrar de
Álgebra Linear (também se ensina um pouco desse tipo de intuição em
cursos de Geometria Algébrica classica).

Abraços
-- 
Bernardo Freitas Paulo da Costa

-- 
Esta mensagem foi verificada pelo sistema de antivírus e
acredita-se estar livre de perigo.


=
Instruções para entrar na lista, sair da lista e usar a lista em
http://www.mat.puc-rio.br/~obmlistas/obm-l.html
=
-- 
Esta mensagem foi verificada pelo sistema de antivírus e
 acredita-se estar livre de perigo.



Re: [obm-l] Problemas interessantes

2013-08-24 Por tôpico Eduardo Wilner
Um triângulo  equilátero de lado nse divide em ntriângulos de lado 1 ???!!!

 




 De: Benedito bened...@ufrnet.br
Para: obm-l@mat.puc-rio.br 
Enviadas: Quinta-feira, 22 de Agosto de 2013 4:39
Assunto: [obm-l] Problemas interessantes
 


Segue dois problemas interessantes.
Benedito
 
Problema 1
Um triângulo equilátero de lado 2012 está dividido em 2012  triângulos 
equiláteros menores de lado 1
mediante paralelas ao seus lados. Em cada vértice de um triângulo menor há uma 
formiga. No mesmo instante, 
todas as formigas começam a caminhar com a mesma velocidade pelas retas da 
triangulação.
Ao chegar  a outro vértice giram 60º ou 120º à esquerda ou à direita e  seguem 
movendo-se.
Determinar se é possível que este movimento se desenvolva para sempre sem ter  
nunca duas
formigas em um mesmo vértice de um triângulo menor.
 
Problema 2
Associar aos vértices de um polígono convexo de 33 lados os números inteiros de 
1 a 33, sem repetir, e em seguida, associar aos lados do polígono a soma dos 
números de seus extremos. 
O objetivo é que os números associados aos lados sejam 33 inteiros consecutivos 
ordenados.
-- 
Esta mensagem foi verificada pelo sistema de antivírus e 
acredita-se estar livre de perigo. 
-- 
Esta mensagem foi verificada pelo sistema de antivírus e
 acredita-se estar livre de perigo.



[obm-l] Re: [obm-l] Re: [obm-l] Re: [obm-l] Re: [obm-l] Divisibilidade(congruência)

2013-07-11 Por tôpico Eduardo Wilner
Leia  a mensagem inicial do Marcone Augusto Araujo Borges.
A questão original perdeu-se pelo caminho (parece o jogador que vai driblando e 
e esquece a bola ou a brincadeira do telefone sem fio, de antigamente, 
claro...)  




 De: Bernardo Freitas Paulo da Costa bernardo...@gmail.com
Para: obm-l@mat.puc-rio.br 
Enviadas: Quinta-feira, 11 de Julho de 2013 15:19
Assunto: [obm-l] Re: [obm-l] Re: [obm-l] Re: [obm-l] 
Divisibilidade(congruência)
 

2013/7/11 Eduardo Wilner eduardowil...@yahoo.com.br:

 De: Lucas Prado Melo luca...@dcc.ufba.br
 Para: obm-l@mat.puc-rio.br
 Enviadas: Quinta-feira, 11 de Julho de 2013 6:43
 Assunto: [obm-l] Re: [obm-l] Divisibilidade(congruência)

  2013/7/11 Artur Costa Steiner steinerar...@gmail.com
 
   O Bernardo já mostrou que m + n é múltiplo de 3. Resta mostrar que é 
   também
   múltiplo de 8. Pelo mesmo raciocínio, mn = -1 (mod 8). Para que isto seja
   possível, um dos números m e n tem que ser congruente a 1 módulo 8 e, o
   outro, congruente a -1. Logo, m + n = 1 + (-1) = 0 (mod 8), ou seja,  m + 
   n
   é múltiplo de 8

 m poderia ser 3 e n ser 5.
 3*5 = 15 = 16 - 1 = -1 (mod 8)

 CUIDADO:  nem 3*5+1=16, nem 3+5=8 é divisível por 24

Não, mas o princípio é analisar módulo cada uma das potências de
primos que dividem 24, e o Artur e o Lucas completaram a solução com a
parte mod 8.

 Aliás, a  propriedade vale par qualquer divisor, desde que seja menor que
 pelo menos um entre m e n .

Que propriedade?


-- 
Bernardo Freitas Paulo da Costa

-- 
Esta mensagem foi verificada pelo sistema de antivírus e
acredita-se estar livre de perigo.


=
Instruções para entrar na lista, sair da lista e usar a lista em
http://www.mat.puc-rio.br/~obmlistas/obm-l.html
=
-- 
Esta mensagem foi verificada pelo sistema de antiv�rus e
 acredita-se estar livre de perigo.



Re: [obm-l] Probabilidade - muito interessante...

2013-07-11 Por tôpico Eduardo Wilner
Consideremos o embarque dos 136 passageiros, inclusive você, i.e. excluindo o 
último (consideramos o voo lotado)
Assim que alguém (inclusive você) ocupar o seu lugar ou o do último passageiro 
a embarcar, os passageiros seguintes encontrarão o próprio lugar vago, 
ocupando-o.
Portanto, quando o centésimo trigésimo sétimo passageiro embarcar, encontrará 
vago ou o próprio lugar ou o seu, com probabilidade meio à meio    




 De: Mauricio de Araujo mauricio.de.ara...@gmail.com
Para: obm-l@mat.puc-rio.br 
Enviadas: Quinta-feira, 11 de Julho de 2013 10:16
Assunto: [obm-l] Probabilidade - muito interessante...
 


Recentemente, eu peguei um avião que tinha 137 assentos. Eu gosto sempre de 
ser o primeiro a embarcar e não foi diferente nesta ocasião. Infelizmente, 
assim que eu entrei no avião, percebi que havia perdido o meu cartão de 
embarque e não conseguia me lembrar de qual era o meu assento. Sem saber o que 
fazer, eu escolhi aleatoriamente um assento qualquer e me sentei. Claro que 
havia a probabilidade de 1/137 de eu ter escolhido o assento correto, ou seja, 
aquele que estava marcado no meu cartão de embarque. À medida que os demais 
passageiros embarcavam, cada um se dirigia ao seu assento e sentava-se, a menos 
que o mesmo estivesse ocupado. Neste caso, o passageiro abria mão de sentar-se 
no assento que estava originalmente atribuído a ele (conforme o cartão de 
embarque) e escolhia um outro assento qualquer para se sentar. Percebi que fui 
o único passageiro que perdeu o cartão de embarque.

A questão que se coloca é a seguinte: qual a probabilidade de o último 
passageiro a embarcar encontrar o seu assento desocupado, ou seja, encontrar o 
assento que está no seu cartão de embarque disponível para ele se sentar?

Este problema está explicado no livro Introduction to counting and 
probability do David Patrick e tem uma resposta surpreendente: a probabilidade 
é de 50%...

Para sentir a solução, vale a pena pensar no problema para os casos em que o 
avião tem 2, 3, 4 e 5 assentos... 
-- 

Abraços

oɾnɐɹɐ ǝp oıɔıɹnɐɯ
momentos excepcionais pedem ações excepcionais.

Os cemitérios estão cheios de pessoas insubstituíveis em seus ofícios.
-- 
Esta mensagem foi verificada pelo sistema de antiv�us e 
acredita-se estar livre de perigo. 
-- 
Esta mensagem foi verificada pelo sistema de antiv�rus e
 acredita-se estar livre de perigo.



[obm-l] Re: [obm-l] Divisibilidade(congruência)

2013-07-10 Por tôpico Eduardo Wilner
A formulação não está correta; contra-exemplo : m=3 e n= 9

[ ]'s





 De: marcone augusto araújo borges marconeborge...@hotmail.com
Para: obm-l@mat.puc-rio.br obm-l@mat.puc-rio.br 
Enviadas: Quarta-feira, 10 de Julho de 2013 22:17
Assunto: [obm-l] Divisibilidade(congruência)
 


 
Sejam m e n dois números naturais tais que mn + 1 é multiplo de 24.
Mostre que m + n tambem é múltiplo de 24.

Se possivel,gostaria que alguem resolvesse por congruencia.Obrigado.
-- 
Esta mensagem foi verificada pelo sistema de antivírus e 
acredita-se estar livre de perigo. 
-- 
Esta mensagem foi verificada pelo sistema de antiv�rus e
 acredita-se estar livre de perigo.



[obm-l] Re: [obm-l] Divisibilidade(congruência)

2013-07-10 Por tôpico Eduardo Wilner
Desculpem, desconsiderem ; confundí 24 com 14 (deve ser o sono às duas da 
madruga...)

Boa noite



 


A formulação não está correta; contra-exemplo : m=3 e n= 9

[ ]'s





 De: marcone augusto araújo borges marconeborge...@hotmail.com
Para: obm-l@mat.puc-rio.br obm-l@mat.puc-rio.br 
Enviadas: Quarta-feira, 10 de Julho de 2013 22:17
Assunto: [obm-l] Divisibilidade(congruência)
 


 
Sejam m e n dois números naturais tais que mn + 1 é multiplo de 24.
Mostre que m + n tambem é múltiplo de 24.

Se possivel,gostaria que alguem resolvesse por congruencia.Obrigado.
-- 
Esta mensagem foi verificada pelo sistema de antivírus e 
acredita-se estar livre de perigo. 
-- 
Esta mensagem foi verificada pelo sistema de antiv�rus e
 acredita-se estar livre de perigo.



Re: [obm-l] Ajuda

2013-05-12 Por tôpico Eduardo Wilner
Faltam condições...





 De: Marcelo de Moura Costa mat.mo...@gmail.com
Para: obm-l@mat.puc-rio.br 
Enviadas: Domingo, 12 de Maio de 2013 12:11
Assunto: [obm-l] Ajuda
 


Determine as raízes da equação aX² + bX + C = 0  sabendo que 4a - 6b + 9c = 0.

Re: [obm-l] Ajuda

2013-05-05 Por tôpico Eduardo Wilner
No texto inicial, a gritante interrogação, se refere ao que? Ao lado?

[ ] s  




 De: Marcelo de Moura Costa mat.mo...@gmail.com
Para: 
Enviadas: Domingo, 5 de Maio de 2013 5:42
Assunto: [obm-l] Ajuda
 


Tenho certeza de que alguém da lista já se deparou com esse problema e sua 
solução:

Um ponto interno de um triângulo equilátero dista 5cm, 7cm e 8cm dos vértices 
do triângulo.?

Solução:

3(p^4 + q^4 + t^4 + a^4) = (p^2 + q^2 + t^2 + a^2)^2.

p = 5
q = 7
t = 8

a=lado do triângulo equilátero. 

Alguém já viu a demonstração ou conhece?
Agradeceria a informação.

Abraços e boa semana.

Marcelo

[obm-l] Re: [obm-l] Sistema de Três Equações com Quadrados

2013-05-05 Por tôpico Eduardo Wilner
Deve haver algum engano: sistema de três equações (parecem L.I.) à duas 
incógnitas?




 De: terence thirteen peterdirich...@gmail.com
Para: obm-l obm-l@mat.puc-rio.br 
Enviadas: Quarta-feira, 1 de Maio de 2013 21:02
Assunto: [obm-l] Sistema de Três Equações com Quadrados
 



Resolva o sistema abaixo:

3(S-l)^2+D^2=3^2
3S^2+(l-D)^2=4^2
3S^2+(l+D)^2=5^2



(Espero que minha formulação esteja correta...)


-- 
/**/
神が祝福

Torres 

[obm-l] Re: [obm-l] Polígono regular inscrito

2013-05-02 Por tôpico Eduardo Wilner
Os fantásticos números complxos resolvem.

Vejamos, como ilustração, o triângulo ( e um pouco do quadrado) que vc. 
resolveu geométricamente

Sejaz = e^(ib)  , 0 =b2pi,  representando os pontos da circunferência com 
centro na origem do plano complexo(Argand-Gauss) .

As raizes de terceira ordem de 1, i.e., da equação z^3=1, representam os 
vértices do triângulo inscrito:

 e^(i3b) =1   = 3b=j.2.pi = b=j.2.pi/3 , j= {0,1 ,2}; assim, os vértices do 
triângulo localizam-se em   


z_0=1, no semi-eixo real positivo, (seja o vértice V_0) , z_1=e^(i2.pi/3)  e  
z_2=e^(i4.pi/3) .

Poderiamos também partir da equação  z^3 - 1 =0 (I)  , e sabendo que uma 
solução é z=z_0=1, dividi-la por z - 1 obtendo, z^2 + z + 1=0 , que reproduz as 
mesmas raizes acima, z_1 e z_2  (z_2 apareceria como e^(-i2.pi/3)=e^(i4.pi/3) ).

Para as cordas, sem perda de generalidade, escolhemos o vértice V_0 como o 
comum à elas e definimos como w = z  - z_0 =z -1 as cordas vetoriais. Fazendo 
a mudança de variável na equação (I), 
   (w+1)^3  - 1 =0  = w^3 + 3w^2 + 3w  = 0 , que dividida por w=w_0= 0 (a 
corda V_0V_0, que gracinha...) resultando em w^2 +3w + 3 =0 cujas raizes são as 
de z subtraidas de 1. Mas , no caso, o que importa é que o termo indepente 
fornece o produto das raizes, 3. ( observe  que parao comprimento das cordas 
teriamos que trocar w por  |w| mas como, no produto há compensação de sinais, 
não é necessário.    


 Para o quadrado mantendo z=z_0=1 e dividindo a equação z^4 - 1= 0 (II)por z-1, 
obtemos a equação
z^3+z^2+z+1=0 , cujas raizesrepresentam os outros 3 vértices (os dois do eixo 
imaginário e o do semi=eixo real positivo. Fazendo a mudança de variável, z = w 
+ 1 em (II),
  (w+1)^4 - 1 =0  = w^4 + 4w^3+6w^2+4w=0, que dividida por w fornece  


   w^3+4w^2+6w+4 = 0, termo independente , produto das raizes, 4.

Genéricamente, para o polígono de n lados, a equação z^n - 1 = 0 (III), em 
termos de w,

(w+1)^n  - 1 = w^n + n w^(n-1) + ...+ (n , j) w^(n-j) +...n=0 , onde (n , j) 
são os coeficientes binomiais, mostra que o termo independente , produto das 
raizes, é n.

[ ]'s  


    




 De: Martins Rama martin...@pop.com.br
Para: obm-l@mat.puc-rio.br 
Enviadas: Quarta-feira, 1 de Maio de 2013 3:53
Assunto: [obm-l] Polígono regular inscrito
 

Caros amigos da lista...
A afirmação abaixo é verdadeira? Como prová-la? Indução, talvez?

Para um polígono regular convexo de n vértices V1, V2, ...,Vn, inscrito
num círculo de raio unitário, qual o valor do produto das medidas das
(n-1) cordas traçadas de um vértice, por exemplo, V1?

P = V1V2 x V1V3 x ... x V1Vn = ?

Sei que para o:
- triângulo equilátero, temos: (raiz de 3)x(raiz de 3) = 3
- quadrado, temos: (raiz de 2)x(raiz de 2)x2 = 4
- hexágono regular, temos: 1x(raiz de 3)x2x(raiz de 3)x1 = 6

É possível generalizar a solução e encontrar a resposta n para todos os
polígonos regulares?

Abraços,
Martins Rama.

=
Instruções para entrar na lista, sair da lista e usar a lista em
http://www.mat.puc-rio.br/~obmlistas/obm-l.html
=

Re:[obm-l]Questoes

2012-11-28 Por tôpico Eduardo Wilner
Me perdí no novo formato. Existe algum botão para postar resposta? E um novo 
subject

Assim, estou tentando à moda antiga = por e-mail...

Fabio Silva pode dividir o quadrilátero em dois triângulos, por exemplo, de 
mesma base 4 e alturas  x.sen 30º=x/2 e  (10-x).sen 30º=(10 - x)/2.

Assim a àrea será

 (1/2).4.(x+10-x)/2= 10.

[ ]'s


Re: [obm-l] Ajuda em geometria

2012-11-04 Por tôpico Eduardo Wilner


Podemos considerar um dos quadriláteros como um quadrado de vértices Qi e 
pontos médios Mi i(1 2 3 4) e o outro convexo qualquer de vértices Gi, que a 
transitividade garante a generalidade.

Devido a convexidade teremos vértices opostos, sejam os de i impar, do quadrado 
no interior ao genérico, seja i=1, e o de i=3, ao contrário: o vértice do 
genérico no interior do quadrado. Os outros vértices, de i par são, de cada 
quadirlátero externo ao outro.

Cada triãngulo QiMiGi é congruente ao Q(i+1)MiG(i+1), até i igual a3 (lógico) e 
completa com Q4M4G4 congruente a Q1M4G1.

Assim, a área do triângulo de i=1, excesso do genérico em relação ao quadrado é 
compensado pelo seu congruente, excesso do quadrado em relação ao gnérico , com 
exceção de um triângulo (seja Q2G2E2) externo à ambos, mas que compensa na 
comparação do par seguinte , i.e., de i=2 com seu congruente. 

E assim repete-se num ciclo, completando as áreas.

[ ]'s 


[obm-l] Re:[obm-l] FW: Tentei e não consegui(geometria)

2012-10-25 Por tôpico Eduardo Wilner
Pela semelhança dos triangulos ABE (base AB=a, altura h2) e CDE(base CD=b, 
altura h3) ,
  (a/h2) = (b/h3) = k.
Assim,  A2 = a*h2/2 = (k/2)(h2)^2  = h2 = \sqrt (2*A2/k) (I)

Analogamente  A3 = (k/2)(h3)^2   =  h3 = \sqrt(2*A3/k)   (II)

 A1 = (h/2)(a+b) = (h/2)k(h2+h3) = (k/2)h^2 (pois  (IV) h2+h3=h) = h 
=\sqrt(2*A1/k) (III)

Substituindo  (I), (II) e (III) em (IV) e simplificando, obtemos a relaçao 
proposta.

[ ]'s


[obm-l] Re:[obm-l] Geometria(Construção(2))

2012-09-11 Por tôpico Eduardo Wilner
Parece que há uma inversão na posição dos pontos, não?

[ ]'s



[obm-l] Re: [obm-l] Re: [obm-l] Socorro em geometria (construçã o)

2012-09-09 Por tôpico Eduardo Wilner
Soh para ficar bem claro: eh por C passe uma paralela a AT.

[ ]'s

--- Em sáb, 8/9/12, Julio César Saldaña saldana...@pucp.edu.pe escreveu:

De: Julio César Saldaña saldana...@pucp.edu.pe
Assunto: [obm-l] Re: [obm-l] Socorro em  geometria (construçã o)
Para: obm-l@mat.puc-rio.br
Data: Sábado, 8 de Setembro de 2012, 8:51



Contrói o triângulo ACT com lados AC=p, CT=q e AT=s.

Por T passe uma paralela a AT. Com centro em A desenhe um arco de raio a. Esse
arco vai cortar à paralela (tem 2 soluções). Chame de D a ese ponto de corte.
Por D traçe uma paralea a CT, o ponto de corte entre essa paralea e AT será o
ponto B do trapecio ABCD procurado.

Julio Saldaña


-- Mensaje original ---
De : obm-l@mat.puc-rio.br
Para : obm-l@mat.puc-rio.br
Fecha : Sat, 8 Sep 2012 03:09:59 +
Asunto : [obm-l] Socorro em  geometria (construçã o)




Construir o trapézio ABCD conhecendo a soma das bases AB +CD = s,as diagonais
AC = p e BD = q e o lado AD = a.Justifique.                        

__
Si desea recibir, semanalmente, el Boletín Electrónico de la PUCP, ingrese a:
http://www.pucp.edu.pe/puntoedu/suscribete/

=
Instruções para entrar na lista, sair da lista e usar a lista em
http://www.mat.puc-rio.br/~obmlistas/obm-l.html
=


[obm-l] Re: [obm-l] Ajuda e orientações

2012-09-04 Por tôpico Eduardo Wilner
Por isso o enunciado coloca se duas parábolas,,,

Uma maneira de provar, usando GA seria escolher, spg, uma parábola com a 
diretriz coincidente com Ox, de foco (a,b) e a outra com a diretriz 
coinicidindo com Oy e foco (c,d), sendo (x,y) qualquer dos quatro pontos de 
intersecção (claro que os focos são tais que a mencionada
 condição ocorra).  

Usando a condição de definição (mesma distância ao foco e à diretriz) para a a 
primeira parábola, temos

    y^2 = (x-a)^2+(y-b)^2   =   x^2 - 2.a.x - 2.b.y = - b^2 - a^2  
(I)

e para a segunda 

                x^2= (x-c)^2+(y-d)^2 =   y^2 -2.d.y - 2.c.x = - c^2 - d^2 
(II)

Somando membro a membro, (I) + (II)

               [x - (a+c)]^2 - (a+c)^2  +  [y - (b+d)]^2 - (b+d)^2 =  - b^2 - 
a^2 - c^2 - d^2

 ou    

   [x - (a+c)]^2  +  [y - (b+d)]^2  =  2(a.c+b.d)

    que relaciona claramente a posição dos focos com o  raio e o centro da 
circunferencia. 


--- Em dom, 2/9/12, Marcelo de Moura Costa mat.mo...@gmail.com escreveu:

De:
 Marcelo de Moura Costa mat.mo...@gmail.com
Assunto: [obm-l] Ajuda e orientações
Para: obm-l@mat.puc-rio.br
Data: Domingo, 2 de Setembro de 2012, 17:27

Foi-me apresentado o seguinte problema:

Mostre que se duas
 parábolas, com retas focais perpendiculares entre si, se
intersectam em quatro pontos, então estes pontos pertencem a um círculo.

O problema começa em que o fato das retas focais serem perpendiculares não 
garante


que haverá 4 pontos de intersecção entre as parábolas, é necessário pelo menos 
que os focos de ambas 
encontrem-se no mesmo quadrante formado pelas perpendiculares e a uma 
determinada distância.
Ou eu estou enganado? Gostaria muito de uma orientação quanto a esse problema.




[obm-l] Re: [obm-l] Ajuda e orientações

2012-09-04 Por tôpico Eduardo Wilner
Em tempo...(?não dá para editar...) Estou procurando uma demonstração mais 
elegante
i.e., sem GA.

[ ]'s


[obm-l] Re:[obm-l] Re: [obm-l] Geometria Plana - Triângulo

2012-08-31 Por tôpico Eduardo Wilner
Carlos Vitor, poderia explicar por que o quadrilatero ACHE eh ciclico?

Vc. estah considerando EH paralelo a AC? Por que?
 
[ ]'s



[obm-l] Re: [obm-l] problema da divisão

2012-08-14 Por tôpico Eduardo Wilner
Me parece que estipulando que aquele que divide é o ultimo a pegar sua parte, 
resolve.

Ou não é este o espírito da questão? 


[ ]'s


Re: RE: [obm-l] Geometria espacial

2012-08-05 Por tôpico Eduardo Wilner
Acho que estamos falando aqui sobre o caso em que o raio das esferaas eh 
máximo, i.e., cada face do tetraedro tangencia tres das esferas.
Assim, Maldonado, seu tetraedro estah muito subdimensionado; vc. soma um r ah 
altura do tetraedro interno na base, OK, mas um r no vertice(?) não estah OK.
Acho
 mais facil considerar que a aresta eh proporcional ao inraio , um 
quarto da altura,  e levar em conta que o inraio do externo eh igual ao do 
interno mais um raio das esferinhas, chegando sem problemas ah 

 a = 2r(1+sqrt6).

Que
 tal considerar o outro extremo; esferas com o minimo raio, i.e. , cada 
face do tetraedro tangenciando apenas uma das esferas?

[ ]'s

Re:Re: [obm-l] Geometria Espacial PIRAMIDE

2012-07-26 Por tôpico Eduardo Wilner
É verdade; eu assumí a reta r passando pelo ponto O...

[ ]'s


Re: [obm-l] Geometria Espacial PIRAMIDE

2012-07-21 Por tôpico Eduardo Wilner
Parece haver algum engano, ou eu não entendí o enunciado

Podemos construir um corte vertical da pirâmide como um triângulo retângulo 
com um cateto sendo a metade da aresta, a/2, a hipotenusa como a altura do 
triângulo equilátero, da face lateral, (a/2) 3^(1/2), portanto o outro cateto, 
altura da pirâmide, (a/2)2^(1/2).

Assim, a distância pedida é a altura, d , desse triângulo 

 d = (a/2)(a/2)2^(1/2)/[(a/2)3^(1/2) = (a/6)6^(1/2)

[ ]'s 


Re:[obm-l] Probleminha

2012-07-18 Por tôpico Eduardo Wilner



Seja X o volume do tonel e x o volume da caneca.
Na primeira operação restou X-x de vinho e x de água.

Admitindo que o cliente agitou bem antes de usar a segunda dose, foi retirado 
(x/X)x de água e reposto x, logo  a quantidade final de água será 2x-(x^2)/X = 
X/2.

Resolvendo, a solução (menor que X) é x = X (2-sqrt2)/2.

Se for dirigir, não beba!

[ ]'s 


[obm-l] Re:[obm-l] A função e^x

2012-06-11 Por tôpico Eduardo Wilner
No item 1) a e b são reais?


[obm-l] RE:[obm-l] Eu não entendo este problema

2012-05-13 Por tôpico Eduardo Wilner







Parece que drechum é estrangeiro... de onde ?Neste 
problema, se bem o entendí ( bem quer dizer médio...) o raio da circunferência 
é igual aos lados do triângulo e do quadrado, que parece ser 2 (?)
[ ]'s


drechum

Sun, 13 May 2012 14:07:02 -0700








ABC é um triângulo eqüilateral; BCDE é um quadrado 2 lateral construído 
exteriormente ao triângulo. Os vértices A, D e E que eles pertencem à mesma 
circunferência. Acha o valor do rádio da circunferência.

Re: [obm-l] Soma

2012-04-24 Por tôpico Eduardo Wilner



Sn = a1.[1 - r^n] / [1 - r] = 2^n - 1 , já que a1 = 1 e r=2 !

A exclamação é exclamação e não fatorial e perdão pelos colchetes já que meu 
gerador de caracteres (ou talvez o teclado) se recusa a fazer o parêntesis.


[ ]s


--- Em ter, 24/4/12, J. R. Smolka smo...@terra.com.br escreveu:

De: J. R. Smolka smo...@terra.com.br
Assunto: Re: [obm-l] Soma
Para: obm-l@mat.puc-rio.br
Data: Terça-feira, 24 de Abril de 2012, 8:43


  

  
  
Obrigado Nehab. Você está certo. Mas, corrigindo isso, o resultado
vai para (n + 1).2^n - 1, e não para o (n - 1).2^n + 1 que outras
pessoas encontraram. Porque?



[ ]'s



J. R. Smolka



Em 23/04/2012 19:21, Carlos Nehab escreveu:

  
  Oi, Smolka,



Na expressão do X - 2X  você se distraiu no sinal do  n.2^n que
é menos. 



Abraços 

Nehab



Em 23/04/2012 16:45, J. R. Smolka escreveu: 
  


 Vejamos...

  

  X = 1.2^0 + 2.2^1 + 3.2^2 + ... + n.2^(n - 1) 

  2X = 1.2^1 + 2.2^2 + 3.2^3 + ... + n.2^n

  

  X - 2X = 1 + (2 - 1).2^1 + (3 - 2).2^2 + ... + [(n - 1) - (n -
  2)].2^(n - 1) + n.2^n 

  -X = 1 + 2 + 2^2 + 2^3 + ... + 2^(n - 1) + n.2^n

  

  Os n primeiros termos do lado direito da equação formam uma PG
  com termo inicial a1 = 1 e razão r = 2. A soma destes n
  primeiros termos da PG é igual a: 

  

  Sn = a1.(1 - r^n) / (1 - r) = 1 - 2^n 

  

  então: 

  

  -X = 1 - 2^n + n.2^n = 1 - (n - 1).2^n == X = (n - 1).2^n
  - 1 

  

  Onde errei, então? 

  

  [ ]'s 

  

  J. R. Smolka 

  

  Em 23/04/2012 13:15, Eduardo Wilner escreveu:

  

  
Quase Smolka,
  


  (n-1)2ˆn +1 .
  


  [ ]`s

 

  

  

  



  



RE: [obm-l] Soma

2012-04-23 Por tôpico Eduardo Wilner
Quase Smolka,
(n-1)2ˆn +1 .
[ ]`s

--- Em seg, 23/4/12, marcone augusto araújo borges 
marconeborge...@hotmail.com escreveu:

De: marcone augusto araújo borges marconeborge...@hotmail.com
Assunto: RE: [obm-l] Soma
Para: obm-l@mat.puc-rio.br
Data: Segunda-feira, 23 de Abril de 2012, 11:15





Obrigado a todos q responderam

Eu tava calculando a soma 1 + (1+2)+ (1 + 2 + 2^2) + ... + ( 1+ ... + 2^(n-1))

Dai veio a dúvida,mas já sei uma maneira mais simples de calcular a soma acima. 
 




Date: Sun, 22 Apr 2012 11:42:49 -0300
Subject: Re: [obm-l] Soma
From: teixeira.discuss.m...@gmail.com
To: obm-l@mat.puc-rio.br


Olá,


Encontrei o seguinte:


2S=        2+2X2^2+3X2^3+4X2^4++(n-1)X2^(n-1)+nX2^n 
--
S=  1+2X2+3X2^2+4X2^3+5X2^3++(n-1)X2^(n-1)              


S= -1-(2+2X2^2+2X2^3+...+2^(n-1))+nX2^n


S= -1-2^n+2+nX2^n


S= (n-1)x2^n+1.


Teixeira.


Em 22 de abril de 2012 08:08, J. R. Smolka smo...@terra.com.br escreveu:

Ops... cometi o velho erro de trocar o sinal. resposta final deve ser 
(n-1).(2^n) - 1
=
Instruções para entrar na lista, sair da lista e usar a lista em
http://www.mat.puc-rio.br/~obmlistas/obm-l.html
=

  


[obm-l] Re: [obm-l] Re: [obm-l] aritmética

2012-04-16 Por tôpico Eduardo Wilner
A idéia seria repetir para a base r2 e eliminar X+Y (ou f1+f2 como no original) 
entre as duas equações, ficando com a diofantina  em r1 e r2...

[  ]'s

--- Em seg, 16/4/12, Bernardo Freitas Paulo da Costa bernardo...@gmail.com 
escreveu:

De: Bernardo Freitas Paulo da Costa bernardo...@gmail.com
Assunto: [obm-l] Re: [obm-l] aritmética
Para: obm-l@mat.puc-rio.br
Data: Segunda-feira, 16 de Abril de 2012, 3:20

2012/4/16 Carlos Nehab carlos.ne...@gmail.com:
 2) Solução
 X = 0,3737...  Y = 0,7373...
 Na primeira base r1:
 (r1^2-1).X = 3r1+7
 (r1^2-1).Y = 7r1+3
 Somando, (r1^2-1)(X+Y) = 10(r1+1), ou seja,
 (r1-1)(X+Y)=10    (A)
 Dai já sabemos que r1-1 = 1, 2, 5 ou 10. Mas r1  7, logo r1 = 11 e X + Y =
 1
Hum, X e Y são frações, certo? Porque então você insiste que X+Y seja
inteiro (única razão que eu vi para que r1 - 1 divida 10) ? Nesse caso
até que dá certo, mas sei lá, podia ser que X+Y = 1/2 (mas teria
talvez que mudar a expressão na base r2)

Abraços,
-- 
Bernardo Freitas Paulo da Costa

=
Instruções para entrar na lista, sair da lista e usar a lista em
http://www.mat.puc-rio.br/~obmlistas/obm-l.html
=


[obm-l] Re: [obm-l] Não consigo provar

2012-04-15 Por tôpico Eduardo Wilner
Desculpem o branco; eh o que acontece quando a gente fica muuito tempo sem 
usar ou sem ler uma determinada notação...

Alias, eh mais comum e conveniente que se use E=exp(i2π/n), pela expressao de 
Euler (que parecer ser sugerido pela letra E).

Aih fica imediato que cada soma parceial representa um vertice do poligono no 
plano dos complexos, com o primeiro vertice na origem e o último também 
fechando a poligonal.

Digo que a notação de Euler eh mais conveniente jah que, assim, E representa um 
operador que produz a rotação de um angulo 2π/n em relação ao lado anterior. 


[ ]'s

--- Em sáb, 14/4/12, Heitor Bueno Ponchio Xavier heitor.iyp...@gmail.com 
escreveu:

De: Heitor Bueno Ponchio Xavier heitor.iyp...@gmail.com
Assunto: [obm-l] Não consigo provar
Para: obm-l@mat.puc-rio.br
Data: Sábado, 14 de Abril de 2012, 19:18

Não estou conseguindo provar o seguinte:
Para todo n-ágono equiângulo de lados a1, a2, ..., aN. Vale a relação:
a1 + (a2)E + (a2)E²+... + (an) E^(n-1) = 0. Onde E=cis(2π/n)


[obm-l] Re: [obm-l] Não consigo provar

2012-04-15 Por tôpico Eduardo Wilner
Soh para ficar mais explicito, o primeiro lado, portanto o segundo vehrtice 
estao no eixo dos reais.

--- Em sáb, 14/4/12, Heitor Bueno Ponchio Xavier heitor.iyp...@gmail.com 
escreveu:

De: Heitor Bueno Ponchio Xavier heitor.iyp...@gmail.com
Assunto: [obm-l] Não consigo provar
Para: obm-l@mat.puc-rio.br
Data: Sábado, 14 de Abril de 2012, 19:18

Não estou conseguindo provar o seguinte:
Para todo n-ágono equiângulo de lados a1, a2, ..., aN. Vale a relação:
a1 + (a2)E + (a2)E²+... + (an) E^(n-1) = 0. Onde E=cis(2π/n)


[obm-l] Re: [obm-l] Re: [obm-l] aritmética

2012-04-15 Por tôpico Eduardo Wilner
Acho que houve algum engano pois encontrei 

10*r2 - 7*r1 = 3  --  r2 =(7*r1 + 3)/10  --  r1=11 , r2=8   --  r1+r2=19.

[ ]'s

--- Em dom, 15/4/12, Pedro Nascimento pedromn...@gmail.com escreveu:

De: Pedro Nascimento pedromn...@gmail.com
Assunto: [obm-l] Re: [obm-l] aritmética
Para: obm-l@mat.puc-rio.br
Data: Domingo, 15 de Abril de 2012, 19:26


Passando pra base decimal temos:
(I) f1=3*r1^(-1)+7*r1^(-2)+3*r1^(-3)+7*r1^(-4)+...
(II) f2=7*r1^(-1)+3*r1^(-2)+7*r1^(-3)+3*r1^(-4)+...

(III) f1=2*r2^(-1)+5*r2^(-2)+2*r2^(-3)+5*r2^(-4)+...
(IV) f2=5*r2^(-1)+2*r2^(-2)+5*r2^(-3)+2*r2^(-4)+...
Somando as equacoes (I) e (II) :
(f2+f1)/10=  r1^-1   +r1^-2  +r1^-3  +r1^-4+...

Somando (III) e (IV):
(f2+f1)/7=r2^-1  +r2^-2  +r2^-3  +r2^-4+...
Assim, como o lado direito das duas equacoes eh uma PG infinita, temos:

(f2+f1)/10=r1^(-1)/(1 - r1^(-1))=1/(r1 - 1)
(f2+f1)/7=r2^(-1)/(1 - r2^(-1))=1/(r2 - 1)
Igualando:
10/( r1 - 1 )=7/(r2 - 2)10*r2 - 20 =7*r1 - 7

10*r2 - 7*r1 = 13
Como r2 e r1 sao inteiros, resolvendo a equacao diofantina :
r2=7*n + 2r1=10*n + 1
Tem a restricao de a base R1 ser maior que 7 ( pois aparece o digito 7) e a 
base R2 ser maior q 5, logo n=1.

Pelas opcoes do enunciado fazendo n=1, r2=9 e r1=11 , logo : R1+R2=20
Acho q eh isso...Abracos, Pedro.

Em 15 de abril de 2012 18:39, Jefferson Franca jeffma...@yahoo.com.br 
escreveu:

Um aluno muito curioso e estudioso(tomara!) me deu esta questão durante uma 
aula semana passada e tentei, tentei e nada!
Será que alguém pode dar um ajuda aí?
















Em
uma base R1 uma fração F1 se escreve como 0,373737... enquanto que uma
fração F2 é escrita como0,737373 . Em outra base R2, a fração F1 é escrita como 
0,252525...  e a fração F2 como 0,525252...A soma R1 + R2 no sistema de
numeração decimal é:

a)
24   b)
22  c) 21   d)
20   
e) 19





[obm-l] Re: [obm-l] Não consigo provar

2012-04-14 Por tôpico Eduardo Wilner
O que significa E = cis(2π/n) ? Se for cos(2π/n) ( e o aN for an...), não vale 
para n maior que 4, não é ?

[ ]'s

--- Em sáb, 14/4/12, Heitor Bueno Ponchio Xavier heitor.iyp...@gmail.com 
escreveu:

De: Heitor Bueno Ponchio Xavier heitor.iyp...@gmail.com
Assunto: [obm-l] Não consigo provar
Para: obm-l@mat.puc-rio.br
Data: Sábado, 14 de Abril de 2012, 19:18

Não estou conseguindo provar o seguinte:
Para todo n-ágono equiângulo de lados a1, a2, ..., aN. Vale a relação:
a1 + (a2)E + (a2)E²+... + (an) E^(n-1) = 0. Onde E=cis(2π/n)


[obm-l] Re: [obm-l] Re: [obm-l] Re: [obm-l] heptágono regular

2012-03-25 Por tôpico Eduardo Wilner
Olá Felipe!

Então era isso que vc, queria dizer com  Qual intervalo que o perímetro de um 
heptágono regular assume estando inscrito numa circunferência de raio 2,5 cm? ?

Não posicionou muito bem a questão, não é...? e não respondeu a minha 
estranheza...?

Agora não consigo entender desigualdade de lados em um heptágono regular???

[ ]s

--- Em dom, 25/3/12, felipe araujo costa faraujoco...@yahoo.com.br escreveu:

De: felipe araujo costa faraujoco...@yahoo.com.br
Assunto: [obm-l] Re: [obm-l] Re: [obm-l] heptágono regular
Para: obm-l@mat.puc-rio.br obm-l@mat.puc-rio.br
Data: Domingo, 25 de Março de 2012, 11:18

Bom dia Érica.Obrigado pela solução. Essa ja havia pensado e queria saber se ha 
uma por desigualdade dos lados.Abraço. Felipe Araujo Costa
Cel: 77430066
E-mail: faraujoco...@yahoo.com.br
faco...@metalmat.ufrj.br
De: Érica Gualberto Pongelupe Giacoia profer...@ig.com.br
 Para: obm-l@mat.puc-rio.br 
 Enviadas: Domingo, 25 de Março de 2012 10:30
 Assunto: [obm-l] Re: [obm-l] heptágono regular
   
Use a lei dos cossenos e calcule a medida do lado x. 
x^2=2,5^2+2,5^2-2*2,5*2,5*cos(360/7)
Depois, basta multiplicar x por 7.
Abração

Em 25 de março de 2012 10:14, felipe araujo costa faraujoco...@yahoo.com.br 
escreveu:

Bom dia.
Preciso de uma ajuda nessa questão. Quero saber se ha uma resolução por 
desigualdade entre os lados do heptagono.Obrigado.


O perímetro do heptágono regular convexo inscrito num círculo de
 raio 2,5, é um número  real que esta entr
 a)14 e 15
b)15 e 16c)16 e 17d)17 e 18
e)18 e 19
Felipe Araujo Costa





-- 
Érica G. P. G.





[obm-l] Re: [obm-l] Re: [obm-l] Re: [obm-l] heptágono regular

2012-03-25 Por tôpico Eduardo Wilner
Em tempo: estava me referindo à sua mensagem geometria de 22 pp. 

--- Em dom, 25/3/12, felipe araujo costa faraujoco...@yahoo.com.br escreveu:

De: felipe araujo costa faraujoco...@yahoo.com.br
Assunto: [obm-l] Re: [obm-l] Re: [obm-l] heptágono regular
Para: obm-l@mat.puc-rio.br obm-l@mat.puc-rio.br
Data: Domingo, 25 de Março de 2012, 11:18

Bom dia Érica.Obrigado pela solução. Essa ja havia pensado e queria saber se ha 
uma por desigualdade dos lados.Abraço. Felipe Araujo Costa
Cel: 77430066
E-mail: faraujoco...@yahoo.com.br
faco...@metalmat.ufrj.br
De: Érica Gualberto Pongelupe Giacoia profer...@ig.com.br
 Para: obm-l@mat.puc-rio.br 
 Enviadas: Domingo, 25 de Março de 2012 10:30
 Assunto: [obm-l] Re: [obm-l] heptágono regular
   
Use a lei dos cossenos e calcule a medida do lado x. 
x^2=2,5^2+2,5^2-2*2,5*2,5*cos(360/7)
Depois, basta multiplicar x por 7.
Abração

Em 25 de março de 2012 10:14, felipe araujo costa faraujoco...@yahoo.com.br 
escreveu:

Bom dia.
Preciso de uma ajuda nessa questão. Quero saber se ha uma resolução por 
desigualdade entre os lados do heptagono.Obrigado.


O perímetro do heptágono regular convexo inscrito num círculo de
 raio 2,5, é um número  real que esta entr
 a)14 e 15
b)15 e 16c)16 e 17d)17 e 18
e)18 e 19
Felipe Araujo Costa





-- 
Érica G. P. G.





Re: [obm-l] geometria

2012-03-22 Por tôpico Eduardo Wilner
Não entendí intervalo...

O período é 7 X 5 X sen(pi/7) cm, ou não é esse o problema?

[ ]'s 

--- Em qui, 22/3/12, felipe araujo costa faraujoco...@yahoo.com.br escreveu:

De: felipe araujo costa faraujoco...@yahoo.com.br
Assunto: [obm-l] geometria
Para: obm-l@mat.puc-rio.br obm-l@mat.puc-rio.br
Data: Quinta-feira, 22 de Março de 2012, 0:45

Preciso de um ajuda.
Qual intervalo que o perímetro de um heptágono regular assume estando inscrito 
numa circunferência de raio 2,5 cm?
Desde já agradeço.

Re: [obm-l] geometria

2012-03-22 Por tôpico Eduardo Wilner



Desculpe; editando:   período -- perímetro.  

--- Em qui, 22/3/12, Eduardo Wilner eduardowil...@yahoo.com.br escreveu:

De: Eduardo Wilner eduardowil...@yahoo.com.br
Assunto: Re: [obm-l] geometria
Para: obm-l@mat.puc-rio.br
Data: Quinta-feira, 22 de Março de 2012, 15:52

Não entendí intervalo...

O período é 7 X 5 X sen(pi/7) cm, ou não é esse o problema?

[ ]'s 

--- Em qui, 22/3/12, felipe araujo costa faraujoco...@yahoo.com.br escreveu:

De: felipe araujo costa faraujoco...@yahoo.com.br
Assunto: [obm-l] geometria
Para: obm-l@mat.puc-rio.br obm-l@mat.puc-rio.br
Data: Quinta-feira, 22 de Março de 2012, 0:45

Preciso de um ajuda.
Qual intervalo que o perímetro de um heptágono regular assume estando inscrito 
numa circunferência de raio 2,5
 cm?
Desde já agradeço.

[obm-l] Re: [obm-l] Re: [obm-l] ajuda com inequações

2012-03-13 Por tôpico Eduardo Wilner
Desculpe, os parêntesis também podem ser ambos negativos, claro, com x-1; mas 
x=3 não satisfaz a inequação.

[ ]'s  

--- Em seg, 12/3/12, tarsis Esau tarsise...@gmail.com escreveu:

De: tarsis Esau tarsise...@gmail.com
Assunto: [obm-l] Re: [obm-l] ajuda com inequações
Para: obm-l@mat.puc-rio.br
Data: Segunda-feira, 12 de Março de 2012, 18:10

Temos: 1/(x+1) - 2/(3x-1)  0 = (x-3)/[(x+1)(3x-1)]  0

e fazendo-se o estudo na reta, temos as soluções

x  -1 ou 1/3x=3



On Mon, Mar 12, 2012 at 5:17 PM, Adilson Francisco da Silva 
adilson...@gmail.com wrote:

Saudações,

Preciso de ajuda com a seguinte desigualdade:

1/(x+1)  2/(3x-1)

Obrigado
Adilson






[obm-l] Re: [obm-l] ajuda com inequações

2012-03-12 Por tôpico Eduardo Wilner
É fácil verificar que as expressões entre parêntesis são ambas positivas, 
portanto

     (1/3)  x  3.

[ ]'s  

--- Em seg, 12/3/12, Adilson Francisco da Silva adilson...@gmail.com escreveu:

De: Adilson Francisco da Silva adilson...@gmail.com
Assunto: [obm-l] ajuda com inequações
Para: obm-l@mat.puc-rio.br
Data: Segunda-feira, 12 de Março de 2012, 17:17

Saudações,

Preciso de ajuda com a seguinte desigualdade:

1/(x+1)  2/(3x-1)

Obrigado
Adilson



Re: [obm-l] Problema

2012-02-08 Por tôpico Eduardo Wilner



A velocidade da nave que viaja pela diagonal eh o triplo da que viaja pela 
aresta, percorrendo uma diastancia \sqrt3 vezes a percorrida pela segunda, 
portanto num intervalo de tempo menor.
Como elas terminam as viagens no mesmo instante t=0, no instante t=-1 ( no 
exemplo da resolução ) , quando a nave alfa estah partindo de B, a nave 
beta ainda não está partindo ou estaria virtualmente num ponto aquem de A, no 
caso (1-\sqrt3)(1,1,1).

Me parece que o problema seria mais Olímpico (e talvez fosse a intenção do 
autor, que se distraiu) se a nave que viaja pela diagonal tivesse velocidade 
\srt3 vezes a que viaja pela aresta, (quando sua pergunta teria a ver, como 
dizem os jovens) e a distancia seria minima no meio do percurso, igual a 
(sqrt2)/2... 

[ ]'s
 

--- Em dom, 5/2/12,
 Henrique Rennó henrique.re...@gmail.com escreveu:

De: Henrique Rennó henrique.re...@gmail.com
Assunto: [obm-l] Problema
Para: obm-l obm-l@mat.puc-rio.br
Data: Domingo, 5 de Fevereiro de 2012, 12:41

Oi, boa tarde.

A solução do problema 1 da primeira fase do nível universitário na eureka 34 
página 60 
(http://www.obm.org.br/export/sites/default/revista_eureka/docs/eureka34.pdf) 
apresenta a função de posição considerando tempos inicial -1 e final 0, sendo a 
função do objeto mais rápido dada por B(t) = (1,1,1) + rq(3)*t*(1,1,1), onde rq 
é a raíz quadrada. Considerando t = -1 na equação, temos B(-1) = (1-rq(3), 
1-rq(3), 1-rq(3)), que é
 diferente da posição inicial (0,0,0). Outra dúvida é como ficariam as funções 
se considerarmos como tempo inicial e final os valores 0 e 1, respectivamente.


Obrigado

-- 
Henrique




Re: [obm-l] Problema

2012-02-06 Por tôpico Eduardo Wilner
A velocidade da nave que viaja pela diagonal eh o triplo da que viaja pela 
aresta, percorrendo uma diastancia \sqrt3 vezes a percorrida pela segunda, 
portanto num intervalo de tempo menor.
Como elas terminam as viagens no mesmo instante t=0, no instante t=-1 ( no 
exemplo da resolução ) , quando a nave alfa estah partindo de B, a nave 
beta ainda não está partindo ou estaria virtualmente num ponto aquem de A, no 
caso (1-\sqrt3)(1,1,1).

Me parece que o problema seria mais Olímpico (e talvez fosse a intenção do 
autor, que se distraiu) se a nave que viaja pela diagonal tivesse velocidade 
\srt3 vezes a que viaja pela aresta, (quando sua pergunta teria a ver, como 
dizem os jovens) e a distancia seria minima no meio do percurso, igual a 
(sqrt2)/2... 

[ ]'s
 

--- Em dom, 5/2/12, Henrique Rennó henrique.re...@gmail.com escreveu:

De: Henrique Rennó henrique.re...@gmail.com
Assunto: [obm-l] Problema
Para: obm-l obm-l@mat.puc-rio.br
Data: Domingo, 5 de Fevereiro de 2012, 12:41

Oi, boa tarde.

A solução do problema 1 da primeira fase do nível universitário na eureka 34 
página 60 
(http://www.obm.org.br/export/sites/default/revista_eureka/docs/eureka34.pdf) 
apresenta a função de posição considerando tempos inicial -1 e final 0, sendo a 
função do objeto mais rápido dada por B(t) = (1,1,1) + rq(3)*t*(1,1,1), onde rq 
é a raíz quadrada. Considerando t = -1 na equação, temos B(-1) = (1-rq(3), 
1-rq(3), 1-rq(3)), que é diferente da posição inicial (0,0,0). Outra dúvida é 
como ficariam as funções se considerarmos como tempo inicial e final os valores 
0 e 1, respectivamente.


Obrigado

-- 
Henrique




Re: [obm-l] Geometria

2011-12-27 Por tôpico Eduardo Wilner
QH = KP é um postulado?

--- Em seg, 26/12/11, Bernardo Freitas Paulo da Costa bernardo...@gmail.com 
escreveu:

De: Bernardo Freitas Paulo da Costa bernardo...@gmail.com
Assunto: Re: [obm-l] Geometria
Para: obm-l@mat.puc-rio.br
Data: Segunda-feira, 26 de Dezembro de 2011, 19:47

2011/12/26 marcone augusto araújo borges marconeborge...@hotmail.com:
    São escolhidos dois pontos P e Q,um cada cateto de um triângulo
 retângulo.As medidas dos comprimentos dos catetos são a e
 b,respectivamente.Sejam K e H as projeções de P e Q,respectivamente,sobre a
 hipotenusa.Determine o menor valor possível para KP + PQ + QH.
Bom, vou dar uma idéia só:
Desenhe o seu triângulo com a hipotenusa na horizontal. Daí, marque os
valores notáveis dos triângulos retângulos:
catetos a, b, hipotenusa c (c^2 = a^2 + b^2)
a altura h
p + q = c as duas partes da hipotenusa de cada lado da altura.

Seja x a distância entre K e o vértice. Ache o valor de KP por
semelhança. Note que QH = KP, logo você pode achar y = distância de H
até o outro vértice do mesmo jeito. Note que PQ = c - x - y. Juntando
tudo, simplificando, usando que cp = a^2 e cq = b^2, você vai chegar
numa fórmula quadrática em x, do tipo M - N^2 x, e daí o mínimo ocorre
quando x é máximo (ou seja, igual a p). Faça as contas de novo e você
deve chegar na sua fórmula.

 O gabarito dá como resposta 2ab/raiz(a^2+b^2)
Pensando um pouco mais, isso é 2*altura (note que ab = ch). Num
problema desses, é bem razoável começar com um chute que o
mínimo/máximo deve estar numa situação limite (nem que seja pra ter
uma idéia) e daí você vê que um caso o valor é esse, no outro é c =
raiz(a^2 + b^2). Que é maior do que isso por MA = MG. Olhando o
argumento acima, você nota que a transformação depende de forma linear
em x (porque tudo é semelhança), logo mínimo e máximo estão nesses
dois opostos. Corolário: se o triângulo for, além de retângulo,
isósceles (ou seja, 45-45-90) todos os valores são os mesmos.

 Agradeço a quem puder ajudar.

Abraços,
-- 
Bernardo Freitas Paulo da Costa

=
Instruções para entrar na lista, sair da lista e usar a lista em
http://www.mat.puc-rio.br/~obmlistas/obm-l.html
=


[obm-l] RE: [obm-l] RE: [obm-l] Questão simples

2011-12-26 Por tôpico Eduardo Wilner
O menor n é mesmo 8. 
n = 13 não satisfaz. Pode ser verificado, por exemplo, seguindo o raciocínio do 
Bernardo Freitas Paulo da Costa (aperfeiçoado pelo  João Maldonado), da 
condição  

 n^2 + n  - 2.m^2 = 0  , 

onde  m natural.   Aplicando a dita fórmula de Bhaskara 

n = [-1 + sqrt(1+8.m^2)]/2 ,

verificamos que  1 + 8.m^2 (portanto, também sua raiz quadrada) é impar ; assim 
é coerente obtermos n natural desta fórmula.

pode-se verificar que o menor natural  m que produz um discriminante quadrado 
(perfeito; não sei o que vem a ser um quadrado imperfeito???...) é 6, 
resultando  

 n = 8.

Para n= 13,  1 + 8.m^2 teria que ser 729 , logo  m^2 = 91... ( talvez tenha 
havido confusão com 81...  
    

 [ ]s.
    

--- Em sex, 23/12/11, LEANDRO L RECOVA leandrorec...@msn.com escreveu:

De: LEANDRO L RECOVA leandrorec...@msn.com
Assunto: RE: [obm-l] RE: [obm-l] Questão simples
Para: obm-l@mat.puc-rio.br  obm-l@mat.puc-rio.br, LEANDRO L RECOVA  
leandrorec...@msn.com
Data: Sexta-feira, 23 de Dezembro de 2011, 15:38

Eu encontrei 13. 

O numero e dado por. 

Y=sqrt((n+1)n)*(n-1)!

Sent from my HTC Touch Pro2 on the Now Network from Sprint®.










-Original Message- 

From: LEANDRO L RECOVA 

Sent: 12/23/2011 4:31:23 PM 

To: obm-l@mat.puc-rio.br 

Subject: [obm-l] RE: [obm-l] Questão simples 


Marcone,

 

Escreva cada termo usando a fatoracao: (n^2-1)=(n+1)(n-1).

 

O resultado sai bem rapido.

 

Saudacoes,

 

Leandro Recova

Los Angeles, California.

 




From: marconeborge...@hotmail.com

To: obm-l@mat.puc-rio.br

Subject: [obm-l] Questão simples

Date: Fri, 23 Dec 2011 13:39:17 +




#yiv662974303 .yiv662974303ExternalClass .yiv662974303ecxhmmessage P
{padding:0px;}
#yiv662974303 .yiv662974303ExternalClass body.yiv662974303ecxhmmessage
{font-size:10pt;font-family:Tahoma;}

Qual é o menor natural n para o qual (2^2 - 1).(3^2 - 1).(4^2 -1)...(n^2 - 1) é 
um quadrado perfeito?

 

Como se diz aqui,essa eu fiz ??no braço``.Fui calculando cada fator e pareando 
os fatores iguais ou seus fatores primos.

Encontrei n=8.Mas deve haver solução mais interessante.Obrigado por qualquer 
esclarecimento,abraço.








Re:[obm-l] Divisores equidistantes

2011-11-21 Por tôpico Eduardo Wilner
Algo está mal colocado; se tomarmos, por exemplo, 1795 = 1 X 3 X 5 X 7 X 17, 
como é que fica?

[ ]s


Re:Re:[obm-l] Divisores equidistantes

2011-11-21 Por tôpico Eduardo Wilner
Foi o que eu disse: algo está mal colocado; não vale para qualquer natural...

[ ]s


Re: [obm-l] Como resolver esta integral?

2011-11-19 Por tôpico Eduardo Wilner
Se o problema é o cálculo da integral de área da função 1/(x^2+y^2) no setor de 
coroa circular, o mais fácil é integrar em polares para encontrar (pi/4) ln 
(sqrt 2).

[ ]'s 


Re: [obm-l] Problemas de Geometria - Morgado II

2011-11-18 Por tôpico Eduardo Wilner
113) Os triângulos formados com as bases, as diagonais e a altura, h, são 
semelhantes, logo b/h = h/a ,ou, h = sqrt (ab).
Assim a área vale sqrt(ab)(a+b)/2.

Quanto ao 249), não tenho a figura...

[]'s



Re: [obm-l] Velocidades!!

2011-11-02 Por tôpico Eduardo Wilner
Enquanto Lucas chegava ao começo da ponte, Pedro chegava à 1/5 do fim, quando o 
trem estava no começo;  se o trem empatou com Pedro no fim, teria que ter uma 
velocidade cinco vezes a dele , 75 km/h. 


Re: [obm-l] A pulga e o elastico

2011-10-06 Por tôpico Eduardo Wilner
Se realmente os saltos são de 1 cm e a esticadas de 1 metro, nunca...

--- Em qui, 6/10/11, J. R. Smolka smo...@terra.com.br escreveu:

De: J. R. Smolka smo...@terra.com.br
Assunto: Re: [obm-l] A pulga e o elastico
Para: obm-l@mat.puc-rio.br
Data: Quinta-feira, 6 de Outubro de 2011, 21:28


  


  
  
Depende... para qual lado o elástico estica? No mesmo sentido ou no
sentido contrário ao deslocamento da pulga?



 [ ]'s



J. R. Smolka



Em 06/10/2011 18:04, Rogerio Ponce escreveu:
Ola' pessoal,

no instante zero, uma pulga inicia uma viagem sobre um elastico
de 1metro, indo de uma extremidade para a outra, dando saltos de
1cm de comprimento a cada segundo.

Entretanto, meio segundo apos o inicio da viagem, o elastico
comeca a sofrer um puxao a cada segundo, de tal forma que ele
estica mais 1 metro a cada puxada.

Assim, com saltos e puxoes intercalados, pergunta-se:

- Quanto tempo levara' a viagem?


  



[obm-l] Re: [obm-l] Limite difícil

2011-09-10 Por tôpico Eduardo Wilner
Podemos até dispensar o clássico senx/x, pois a substituição trigonométrica 
leva à

c^2( sec x -1)/(c^2.tg^2(x)) = (1 - cos x).cos^2x/(1-cos^2(x)) = 
cos^2(x)/(1+cosx)

cujo li9mite, para x -0 é 1/2.
 
--- Em sáb, 10/9/11, Carlos Nehab ne...@infolink.com.br escreveu:

De: Carlos Nehab ne...@infolink.com.br
Assunto: Re: [obm-l] Limite difícil
Para: obm-l@mat.puc-rio.br
Data: Sábado, 10 de Setembro de 2011, 8:31


  


  
  
Oi, João.



Seu limite tem forte apelo geométrico, pois extrair a raiz
quadrada de  soma de quadradaos remete para triângulos
retângulos...(catetos c e v).

Assim, uma simples troca de variável resolve o problema sem
necessidsde de recursos adicionais além do limite clássico senx/x
tende a 1 qdo x tende a zero...

Faça v = c.tg(teta) e seu limite se tornará trivial, na variável
teta.



Nehab



Em 7/9/2011 20:22, João Maldonado escreveu:

  
  
Como posso provar que o limite:









c(   ( v^2 + c^2) ^(1/2) - c    )/v^2  = 1/2,  quando
  v- 0?






[]s
João
  



  



[obm-l] Re: [obm-l] Integral difícil

2011-07-10 Por tôpico Eduardo Wilner
O problema de sinal é delicado. Devemos tomar cuidado com a convenção do ângulo 

alpha, que agora tomo medido da base no sentido anti-horário:

-  (1/R).v.(dv)/(dw) = g.sen w + u.g.cos w + u.v^2/R ,

que se parece mais com a (sua?) versão do jaumzaum indicada no link doSammyS.

Digo parece pois há a diferença, p.ex., do sinal negativo no primeiro membro.

Curioso que para u = 0,5 pode-se resolver facilmente a eq. dif. mas dá um 
problemão para alfa = 0 ...



--- Em sex, 8/7/11, João Maldonado joao_maldona...@hotmail.com escreveu:

De: João Maldonado joao_maldona...@hotmail.com
Assunto: [obm-l] Integral difícil
Para: obm-l@mat.puc-rio.br
Data: Sexta-feira, 8 de Julho de 2011, 21:55






Boa Tarde a todos
Recentemente postei uma integral que não consegui resolver no fórum  
PHYSICSFORUMS mas não obtive nenhuma resposta satisfatória.O problema a seguir 
é uma preparação para a IPhO, embora só a parte matemática interesse
http://www.physicsforums.com/showthread.php?t=512186
Reduzi o problema a equação encontrada no link acima, queria achar a função de 
velocidade em função da distância, S.   




[obm-l] Re: [obm-l] Integral difícil

2011-07-09 Por tôpico Eduardo Wilner
O problema de número de variáveis pode se resolvido se escrevermos.
(chamando alfa de w = s/R)
a = [(dv)/(R.dw)].v    ou (1/R).v.(dv)/(dw) = g.sen w - u.g.cos w - u.v^2/R ,

onde temos v como função de w=alfa (parece que vc, é o jaumzaun ? que 
enganou-se um pouco com os sinais).

Agora o problema é resolver a equação diferencial ...

[ ]s

--- Em sex, 8/7/11, João Maldonado joao_maldona...@hotmail.com escreveu:

De: João Maldonado joao_maldona...@hotmail.com
Assunto: [obm-l] Integral difícil
Para: obm-l@mat.puc-rio.br
Data: Sexta-feira, 8 de Julho de 2011, 21:55






Boa Tarde a todos
Recentemente postei uma integral que não consegui resolver no fórum  
PHYSICSFORUMS mas não obtive nenhuma resposta satisfatória.O problema a seguir 
é uma preparação para a IPhO, embora só a parte matemática interesse
http://www.physicsforums.com/showthread.php?t=512186
Reduzi o problema a equação encontrada no link acima, queria achar a função de 
velocidade em função da distância, S.   




[obm-l] Re: [obm-l] Re: [obm-l] Re: TEM SOLUÇÃO?

2011-06-16 Por tôpico Eduardo Wilner
Gabriel parece estar considerando preços com precisão de um centavo, mas 
encotramos produdos com frações deste (ex.: combustível),

Porquê Gauss coloca que y = 2,50 e x = 1,91 não serve ?

[ ]'s

--- Em qua, 15/6/11, Gabriel Dalalio gabrieldala...@gmail.com escreveu:

De: Gabriel Dalalio gabrieldala...@gmail.com
Assunto: [obm-l] Re: [obm-l] Re: TEM SOLUÇÃO?
Para: obm-l@mat.puc-rio.br
Data: Quarta-feira, 15 de Junho de 2011, 13:41

Um preço de um produto é da forma k/100, onde k é natural.
Então se considerarmos os preços dos produtos como x/100, y/100, z/100 e w/100 
tem-se o seguinte sistema de variáveis inteiras:
x+y+z+w=711
x*y*z*w=71100


Fazendo um programa para analisar todas as soluções possíveis, a única solução 
inteira é (120,125,150,316) sem contar suas permutações. Então o problema tem 
solução e ela é única.

Eu não consegui fazer sem o computador, acho que não tem um método direto, tem 
de ir eliminando os casos, você pode começar falando que x, y, z e w são 
divisores de 71100 e são menores que 711, mas ainda assim vai ter uns 60 
valores diferentes possíveis para as incógnitas


Gabriel Dalalio

Em 15 de junho de 2011 12:14, Prof. Vitório Gauss vitorioga...@uol.com.br 
escreveu:

Sim. Infinitas soluções.

Pois teremos um sistema possível e indeterminado, já que envolve quatro 
variáveis x, y, z e h, bem como duas equações:

x+y+z+h = 7,11

x.y.z.h= 7,11

Não há outra maneira, senão dar bons valores para as variáveis livres h e z.

Vamos supor que h =1,2 e z=1,5:

Assim...

1,5+1,2+x+y = 7,11
1,5.1,2.x.y = 7,11

Ou seja...


x + y = 4,41

1,8xy = 7,11 -- xy=3,95

Logo, y(4,41-y)= 3,95 

y^2 - 4,41y + 3,95 = 0

y= 3,16 e x = 1,25 

ou 
y = 2,50 e x = 1,91 (não serve)


Desta forma, teríamos a 4-upla (x;y;z;h) como solução: (1,25;3,16;1,50;1,20) 

Abraços.

 



Em 15/06/2011 08:33, Marcelo Costa  mat.mo...@gmail.com  escreveu:
UM ALUNO ME APRESENTOU O SEGUINTE PROBLEMA (RESUMINDO):

UM INDIVÍDUO FUI NUMA LANCHONETE E CONSUMIU 4 SALGADOS DISTINTOS, PEDIU A CONTA 
E PERCEBEU QUE O CAIXA MULTIPLICOU OS PREÇOS E DEU O TOTAL DE R$7,11. ENTÃO 
PEDIU PARA QUE ELE SOME OS PREÇOS E NÃO MULTIPLIQUE, PARA A SUA SURPRESA, DEU O 
MESMO VALOR. QUAL É O PREÇO DE CADA SALGADO?


  


=
Instru�ões para entrar na lista, sair da lista e usar a lista em
http://www.mat.puc-rio.br/~obmlistas/obm-l.html
=




[obm-l] Re: [obm-l] FW: [obm-l] Dúvida em Geometria

2011-06-09 Por tôpico Eduardo Wilner
É uma boa oportunidade de aplicar vetores; o produto escalar dos versores das 
normais às faces fornece o oposto do cosseno do ângulo diedro por elas formado 
(oposto porquê o ângulo entre elas é suplementar ao ângulo diedro).
Considerando um sistema de coordenadas cartesianas com origem no centro da base 
da piramide , eixo Oz contendo a altura e Ox e Oy perpendiculares aos lados do 
quadrado da base pertencentes às faces em pauta,(sejam AB e BC)  o produto 
vetorial

                                            | i     j     k |    
   (A-V) X (B-V) =  | 4   -4    -2| = 16 i +32 k = 16*sqrt(5) n1
    | 4    4    -2|

 onde n1 é o versor da normal à face BVA.

Uma rotação de 90º em torno de Oz fornece a normal à face BVC:

                                          n2 =   ( 16 j +32 k ) /[16*sqrt(5)].

Logo  o cosseno do ângulo diedro = - n1 . n2 = -(32*32)/(16*16*5) = - 4/5.

[ ]'s
 


--- Em dom, 29/5/11, João Maldonado joao_maldona...@hotmail.com escreveu:

De: João Maldonado joao_maldona...@hotmail.com
Assunto: [obm-l] FW: [obm-l] Dúvida em Geometria
Para: obm-l@mat.puc-rio.br
Data: Domingo, 29 de Maio de 2011, 17:30





 


From: joao_maldona...@hotmail.com
To: obm-l@mat.puc-rio.br
Subject: RE: [obm-l] Dúvida em Geometria
Date: Sun, 29 May 2011 01:19:37 -0300



 
 

#yiv285393420 .yiv285393420ExternalClass .yiv285393420ecxhmmessage P
{padding:0px;}
#yiv285393420 .yiv285393420ExternalClass body.yiv285393420ecxhmmessage
{font-size:10pt;font-family:Tahoma;}




Bom, sou  estudante de ensino medio, logo minha resposta pode estar  errada :D
Fazendo z1, z2, z3, z4 como os vértices da base e z5 como o  vértice da 
pirâmide/O como o centro da base, 
O cosseno do ângulo da base é fácil calcular, já que Oz1 = 4sqrt(2) e Oz5 = 2, 
z1z5 = 6, logo cos(z5z1O) = 4sqrt(2)/6 = 2sqrt(2)/3
Para calcular o cossendo do ângulo entre duas faces laterais, primeiramente se 
deve usar o seguinte fato: para calcular o angulo diedral deve-se construir um 
plano perpendicular aos outros 2 planos formadores do diedro, o que coloca o 
diedro em uma posição  2d, no que é fácil de calcular. 
Logo partindo de z2 e z4 faremo  2 retas que encontram  perpendicularmente a 
reta  z1O no ponto P. z2z1P = z4z1P = x podemos calcular facilmente traçando 
Uma reta que parte de z5  e encontra o ponto médio M de z1z2, por  pitágoras 
z5M = 2 sqrt(5), logo  sen(x) = sqrt(5)/3. Deste  modo calculemos z2P 
=8sqrt(5)/3 = z4P. Do triângulo z2Pz4, vemos que cos o angulo  desejado é  -4/5
[]'sJoão

Date: Sat, 28 May 2011 15:37:19 -0700
From: paulobarc...@yahoo.com.br
Subject: [obm-l] Dúvida em Geometria
To: obm-l@mat.puc-rio.br

prezados,
 
Desculpe a dúvida, mas estou encontrando dificuldade num problema bem 
elementar, e peço uma orientação , é o seguinte:
 
Qual o cosseno do ângulo diedro entre duas faces de uma piramide quadrangular 
de altura dois e aresta da base igual a 8..Desde já agradeço.
 
Paulo 


[obm-l] Enc: Re: [obm-l] FW: [obm-l] Dúvida em Geometria

2011-06-09 Por tôpico Eduardo Wilner
Estou repetindo a mensagem pois o que apareceu na lista está muito 
deformado em relação ao que eue enviei antes; os simbolos vetoriais 
devem estar em negrito, que talvez o copilador não aceite. Mas o 
determinante saiu todo desmontado... 

--- Em qui, 9/6/11, Eduardo Wilner eduardowil...@yahoo.com.br escreveu:

De: Eduardo Wilner eduardowil...@yahoo.com.br
Assunto: Re: [obm-l] FW: [obm-l] Dúvida em Geometria
Para: obm-l@mat.puc-rio.br
Data: Quinta-feira, 9 de Junho de 2011, 17:01

É uma boa oportunidade de aplicar vetores; o produto escalar dos versores das 
normais às faces fornece o oposto do cosseno do ângulo diedro por elas formado 
(oposto porquê o ângulo entre elas é suplementar ao ângulo diedro).
Considerando um sistema de coordenadas cartesianas com origem no centro da base 
da piramide , eixo Oz contendo a altura e Ox e Oy perpendiculares aos lados do 
quadrado da base pertencentes às faces em pauta,(sejam AB e BC)  o produto 
vetorial

    | i    j     k |   
(A-V) X (B - V ) = |4   -4   -2| = 16 i + 32 k = 16.sqrt(5) n1
    |4    4   -2|

 onde n1 é o versor da normal à face BVA (V é o vertice superior).

Uma rotação de 90º em torno de Oz fornece a normal à face BVC:

                                          n2 =   ( 16 j +32 k ) /[16*sqrt(5)].

Logo  o cosseno do ângulo diedro = - n1 . n2 = -(32*32)/(16*16*5) = - 4/5.

[ ]'s
 







  


[obm-l] Re: [obm-l] Re: [obm-l] RE: [obm-l] Questão de Química

2011-06-05 Por tôpico Eduardo Wilner
 f) estaria correta se não falasse em massa atômica; o número de massa é a soma 
dos números de prótons e neutrons.  

--- Em dom, 5/6/11, Pierry �ngelo Pereira pierryang...@gmail.com escreveu:

De: Pierry �ngelo Pereira pierryang...@gmail.com
Assunto: [obm-l] Re: [obm-l] RE: [obm-l] Questão de Química
Para: obm-l@mat.puc-rio.br
Data: Domingo, 5 de Junho de 2011, 16:41

Obrigado João, ficou bem claro.

Abraço.

2011/6/5 João Maldonado joao_maldona...@hotmail.com






Olá, 

a) Verdadeira - Prót ons e Nêutrons no núcleo, elétrons na eletrosferab) 
Verdadeira - Bohr provou issoc) Verdadeira -  Ânion é um íon negativo (mais 
elétrons de prótons), Cátion é um íon   positiv (mais prótons que elétrons)
d) Verdadeirae) Falsa (o Hidrogênio por exemplo tem 1 próton mas nenhum nêutron 
- claro que existem outros  isômeros, mas o principal tem essa configuração)f) 
Depende, se considerarmos que o elétron tem  massa 0, isso é verdade. Mas é 
incorreto afirmar que o elétron tem  massa 0, já que TODA partícula tem massa, 
até mesmo a energia tem massa, como foi  provado pela famosa fórmula de einsten 
E=mc². O elétron tem massa praticamente 0, ou seja, despresível. O correto 
seria afirmar que o A é praticamente igual  a massa, mas não  exatamente igual. 
Até  porque nem o  nêutron tem massa igual a do próton   , mas superior.

[]'sJoão

Date: Sun, 5 Jun 2011 13:12:41 -0300
Subject: [obm-l] Questão de Química
From: pierryang...@gmail.com

To: obm-l@mat.puc-rio.br

Pessoal sei que o fórum não é de química, mas acredito que existem bons 
químicos aqui rs. 

Essa questão é do livro Química 1 do Feltre (sessão desafios).


Acredito que a questão não existe apenas uma questão correta, ou seria apenas 
erro na interpretacão?


(UFPR) Para interpretar a grande maioria dos processos químicos, é suficiente 
considerar o átomo como sendo constituído
por apenas três partículas: o próton, o nêutron e o elétron. Esssas três 
partículas não estão distribuídas ao acaso; elas 


interagem entre si e essa interação produz um conjunto organizado, que é o 
átomo.

A respeito do átomo, é correto afirmar:

a) Prótons e nêutrons são encontrados no núcleo, que é a parte do átomo com 
carga elétrica positiva e que 


contém praticamente toda a massa do átomo.

b) Os elétrons, partículas de carga elétrica negativa, distribuem-se em torno 
do núcleo em diversos níveis e subníveis 
energéticos (camadas e subcamadas).

c) Se o número de elétrons em um átomo for igual ao número de prótons, o átomo 
será neutro; se for maior, será 


ânion; se for menor, será cátion.

d) O número de prótons de um átomo é denominado número atômico e é representado 
pela letra Z.

e) O núcleo dos átomos será sempre formado por igual número de prótons e 
nêutrons.



f) A soma dos prótons e nêutrons de um átomo é conhecida como número de massa, 
que é representado pela
letra A e é igual à sua massa atômica.


Abraço, 

Pierry A. Pereira

  




[obm-l] Re:[obm-l] Geometria - qual a menor distância

2011-05-04 Por tôpico Eduardo Wilner
Esta eh a lei da reflexão, na optica, e a demonstração mais simples é a do 
Teixeira.

[ ]'s


[obm-l] Re:[obm-l] Isolar z em função de x?

2011-04-28 Por tôpico Eduardo Wilner
Considere como uma equação do segundo grau em z e aplique Bhaskara.

[ ]'s


[obm-l] Re:[obm-l] Demonstração de somatório

2011-03-03 Por tôpico Eduardo Wilner
A soma dos m/2 primeiros pares ( de 2 à m) ou dos (m+1)/2 impares (de 1 à m) é 
dada por 

[m+2)(m+1)m]/6.   Assim, seu somatório, para n par será 

[(n+1)n(n-1) - (n+2)(n+1)n]/6 = (n-1-n-2)n(n+1)/6 = -n(n+1)/2 

(onde para os impares m=n-1), e para n impar

 [(n+2)(n+1)n - (n+1)n(n-1)]/6 = [(n+2-n+1)(n+1)n]/6 = n(n+1)/2 .


[ ]'s  



  

[obm-l] RE: [obm-l] [obm-l] Geometria Analítica

2011-02-25 Por tôpico Eduardo Wilner







Talvez não seja a solução mais elegante , mas enquanto 
isso...Seja r = |AC| e 2B o ângulo que r forma com Ox.Lei dos senos: sen 3B = 3 
(sen B}/r  ou  r = 3 / (3 - 4 sen² B) ;

Adotemos o parâmetro t = sen² B :

x = r(1-2t)=3(1-2t)/(3-4t)    y² =4r²t(1-t)=36 t(1-t)/(3-4t)². 

Eliminado o parâmetro t obtemos a equação da elipse

(x-2)² - y²/3 = 1 .



  


Date: Thu, 24 Feb 2011 15:15:59 -0300
Subject: [obm-l] [obm-l] Geometria Analítica
From: cortes...@gmail.com
To: obm-l@mat.puc-rio.br

Os extremos da base de um triângulo são A(0,0) e B(3,0). Determinar a 
equação do lugar geométrico do vértice oposto C se este se move de 
maneira que o ângulo da base CAB é sempre igual a duas vezes o ângulo da
 base CBA.


  

[obm-l] RE: [obm-l] [obm-l] Geometria Analítica

2011-02-25 Por tôpico Eduardo Wilner
Desculpem: Hiperbole . 

--- Em sex, 25/2/11, Eduardo Wilner eduardowil...@yahoo.com.br escreveu:

De: Eduardo Wilner eduardowil...@yahoo.com.br
Assunto: RE: [obm-l] [obm-l] Geometria Analítica
Para: obm-l@mat.puc-rio.br
Data: Sexta-feira, 25 de Fevereiro de 2011, 12:02








Talvez não seja a solução mais elegante , mas enquanto 
isso...Seja r = |AC| e 2B o ângulo que r forma com Ox.Lei dos senos: sen 3B = 3 
(sen B}/r  ou  r = 3 / (3 - 4 sen² B) ;

Adotemos o parâmetro t = sen² B :

x = r(1-2t)=3(1-2t)/(3-4t)    y² =4r²t(1-t)=36 t(1-t)/(3-4t)². 

Eliminado o parâmetro t obtemos a equação da elipse

(x-2)² - y²/3 = 1 .



  


Date: Thu, 24 Feb 2011 15:15:59 -0300
Subject: [obm-l] [obm-l] Geometria Analítica
From: cortes...@gmail.com
To: obm-l@mat.puc-rio.br

Os extremos da base de um triângulo são A(0,0) e B(3,0). Determinar a 
equação do lugar geométrico do vértice oposto C se este se move de 
maneira que o ângulo da base CAB é sempre igual a duas vezes o ângulo da
 base CBA.




   


  

Re:[obm-l] Dificuldade numa integral

2010-12-12 Por tôpico Eduardo Wilner
Parece que Maldionado esqueceu o expoente no denominador da integranda...



  

Re: [obm-l] Teste de QI

2010-12-02 Por tôpico Eduardo Wilner
 A propósito a resposta da primeira pergunta era 8,08 - 9 ped.

Com esta resposta o QI cai para 60 pois o número de pedreiros é o dobro...



  

Re: [obm-l] Material com provas do IME

2010-11-16 Por tôpico Eduardo Wilner
Não poderiamos calcular diretamente, sem as rotações, a distância do foco dado, 
(origem), à diretriz dada, 

D = |3*0+4*0-25| / sqrt(3^2+4^2) = 5 = c (1 +ou- e^2) / e^2  ===  c = 5 / (4 
+ou- 1) == 

==2c = 10 / (4 +ou- 1),   como a distância focal ( aquí, +ou- não signifca 

aproximadamente, seria ~  hehehehe, mas sim duplo sinal considerando a 
diretriz do lado oposto ou do mesmo lado em relação ao foco, respectivamente ) ?
Na reta, y = (4/3)x, suporte do eixo maior (e não como tinha sido tomada como a 
outra diretriz, não só porque esta deve ser paralela à primeira, mas também não 
pode conter o foco, origem) , teremos o(s) outro(s) foco(s) à distância 2c da 
origem dado(s) pelo sistema
       x^2+ y^2 = 100 / (4 +ou- 1)^2      

       y = (4/3)x
   Obteriamos, para a primeiro caso (6/5 , 8/5) e o mencionado ( -2 , -8/3) 
para o segundo.
  Os plurais , entre parênteses, pretendem ser uma resposta à observação bem 
humorada Quantos focos tem esta elipse?, na mesma linha do numa boa como 
dizem os jovens. E olha que não tem mais focos devido a excentricidade ser 1/2, 
me parece...Aliás, com boa vontade, poderiamos interpretar como sendo esta a 
intenção da pergunta na questão... 

   Me parece mais simples ou pelo menos mai direto.  
Claro que também teriamos que encontrar outras duas diretrizes, impondo que sua 
equação seja     3x+4y + B = 0, paralela à dada, determinado B pela distância 
ao(s) outro(s) focos 

(18/5 + 32/5 + B) /5  = 5  =B = 15  para o primeiro caso e

(- 6 - 32/3 + B) / 5 = 15/3   B = 125/3    para o segundo

[ ]'s






Sergio Lima Netto

Fri, 12 Nov 2010 03:14:48 -0800








oi Fernando,

Você tem razão. A solução está toda incorreta.

Fazendo uma rotação em torno da origem
dos eixos cartesianos, de modo
a alinhar o novo eixo y' das ordenadas com a reta
diretriz, o foco dado continua na origem e
a diretriz dada passa a ser descrita por x' = 5.

Esta rotação é de um ângulo Q com linhas trigonometricas
tg Q = 3/4, sen Q = 3/5, cos Q = 4/5.

Sabendo que a distância da diretriz ao foco
(assumindo que a diretriz dada corresponde ao foco na origem)
é (b^2/c)  (*), tem-se:

  e = c/a = 1/2
  b^2/c = 5

De onde se conclui (com um algebrismo mínimo)
que a = 10/3, b = 5.sqrt(5)/3 e c = 5/3.

Logo, no novo sistema de coordenadas:
o centro da elipse está em x' = -c = -5/3,
o outro foco está em x' = -2c = -10/3
e a outra diretriz é descrita por x' = -2c - b^2/c = -25/3.

Voltando para o sistema de coordenadas original,
o outro foco está na posição
  x0 = -10/3 sen Q = -2
  y0 = -10/3 cos Q = -8/3
e a outra diretriz (paralela à primeira, como você bem observou)
é descrita por 3x + 4y = d, onde d é dado por
d = 3((-25/3)/sen Q) = -125/3.

Vou corrigir esta solução. Obrigado por apontar este problema no material.

Abraço,
sergio



  

[obm-l] Re:[obm-l] Re: [obm-l] Produtos notáveis e f atoração

2010-11-02 Por tôpico Eduardo Wilner
Roger, não seria

x^2+1/x^2=1  ?



  

Re: [obm-l] ajuda

2010-10-31 Por tôpico Eduardo Wilner
Você deve achar as posições mais próximas de P1 e P2 (sabe como?) que devem ser:

P1(2 , 2, 3)  e  P2(1, 3, 2).

Fazendo P1 = O1 + v1*t  obtem-se o instante t=1 em que a primeira partícula 
chega na sua  posição P1.  O1(1, 1, 3) claro.

Impondo que neste instante a posição da segunda partícula sej P2, encontra-se a 
posição inicial desta, que deve ser O2(1, 2, 1).

[ ]'s

  



  

[obm-l]Re: charada do cineclube

2010-10-24 Por tôpico Eduardo Wilner
Escrevendo 5x+2y+50-0,5x-0,5y=200 sai direto (com x e y inteiros).

[ ]'s



  

[obm-l] Re:[obm-l] Dúvida-Geometria ana lítica

2010-10-11 Por tôpico Eduardo Wilner
Claro! Desculpe a distração anterior.

Vc. pode considerar que a mediana só pode medir 3 se for perpendicular à AB. 
Assim, C será a intersecção do prolongamento de BD com a  paralela à AB (eixo 
dos x) à uma altura 6, sendo A, do triângulo original (o  que vc. mencionou 
como reto è o ânguloBAD), obtuso. 

[ ]s



  

[obm-l] Re: [obm-l] Dúvida-Geometria ana lítica

2010-10-10 Por tôpico Eduardo Wilner
Pelo menos na geometria euclidiana esse triângulo inexiste: a altura relativa à 
AB vale 6, logo o pé da referida mediana está à uma altura de 3, logo sua 
medida tem que ser maior... 



  

Re:[obm-l] Geometria OLIMPIADA

2010-10-01 Por tôpico Eduardo Wilner
1) Deve haver mais uma correção: o segmento EF não intersepta a diagonal BD 
pois é paralelo à mesma.  Admití : em lugar de EF leia-se AF.

Assim, por semelhança dos tirângulos BCD e ECF concluímos EF = 1/2.

Prolongando a reta r que contém EF, nos dois sentidos de tal modo que os 
prolongamentos dos lados AB e AD encontrem r em M' e N' (AM' = (3/2)*AB e 
AN'=(3/2)AD), construimos os pares de triângulos semelhantes 
(M'AF,DAN),(FAE,NAM) e (EAM',MAB) ficando evidente que MN=x= (2/3)EF = 1/3.

2) Não vejo porque o problema pede 1 - x^2= 8/9  ou  1 - (1/x)^2 = -8 , se 
obtemos diretamente o valor de x? Será que existe uma maneira  simples de obter 
diretamente 
1 - (1/x)^2 ?

[ ]'s



  

[obm-l] Bravos Bouskela

2010-10-01 Por tôpico Eduardo Wilner
Muito bem Albert. Assino embaixo e me apresso ( com apreço, hehehe) em corrgir 
meu intersepta: leia-se intercepta (deve ser influência do inglês...

[ ]'s



  

  1   2   3   4   >